RD neurology: formatted Flashcards

1
Q
  1. Features of pilocytic astrocytoma (which is false?)

a. The most common posterior fossa tumour in children
b. Calcify more commonly than medulloblastoma
c. Usually a less aggressive lesion
d. Usually more cystic than solid
e. Show enhancement after contrast administration

A

b. Calcify more commonly than medulloblastoma F JPA < 10% calcify, while 10-20% of MB’s calcify (NeuroReq); Osborn says up to 20% of MB calcify, while says “uncommon” in JPA. Ependymoma calcifies in up to 80%.
1. Features of pilocytic astrocytoma (which is false?) (SK)ANSWER:

a. The most common posterior fossa tumour in children T (although some may MB is); peak age 8-13 years. In adults mets & haemangioblastoma.
* LW: Per gospel of Donnelly: Pilocytic astrocytoma are most common posterior fossa tumour in tumour.

b. Calcify more commonly than medulloblastoma F JPA < 10% calcify, while 10-20% of MB’s calcify (NeuroReq); Osborn says up to 20% of MB calcify, while says “uncommon” in JPA. Ependymoma calcifies in up to 80%.
* LW: agree this is incorrect. Donelly states unlike medulloblastoma and ependymomas, pilocytic astrocytoma usually do not demonstrate calcification or haemorrhage

c. Usually a less aggressive lesion: T WHO grade I tumour
d. Usually more cystic than solid T typically well-circumscribed cyst-like masses with a discrete mural nodule – one-third are microcystic or solid, esp. in older patients
e. Show enhancement after contrast administration T solid portion (cyst with enhancing mural nodule)

How well did you know this?
1
Not at all
2
3
4
5
Perfectly
2
Q
  1. An 11yo girl presents with headache and on MRI an intraventricular tumour of the 4th ventricle is shown. The MOST likely tumour is?
    a. Choroid plexus papilloma
    b. Oligodendroglioma
    c. Ependymoma
    d. Haemangioblastoma
    e. Medulloblastoma
A
  1. An 11yo girl presents with headache and on MRI an intraventricular tumour of the 4th ventricle is shown. The MOST likely tumour is?
    ANSWER:e. Medulloblastoma T arise from roof of 4th ventricle (cerebellar vermis projecting into 4th ventricle); older age than ependymoma but usually < 10 years (but range 5-15 years); 2nd most common PF tumour in kids (after JPA)

a. Choroid plexus papilloma F in kids more common in lateral ventricle
b. Oligodendroglioma F
c. Ependymoma F peak age 1-5 years, 3rd most common PF tumour in kids
d. Haemangioblastoma F rare in children without VHL
e. Medulloblastoma T arise from roof of 4th ventricle (cerebellar vermis projecting into 4th ventricle); older age than ependymoma but usually < 10 years (but range 5-15 years); 2nd most common PF tumour in kids (after JPA)

How well did you know this?
1
Not at all
2
3
4
5
Perfectly
3
Q
  1. Cortically based tumour:
    a. Low grade glioma
    b. DNET
    c. Pilocytic astrocytoma
    d. Cavernous haemangioma
    e. Ependymoma
A
  1. Cortically based tumour:
    ANSWER:b. DNET - T - benign, focal intracortical mass superimposed on background of cortical dysplasia. Arises from secondary germinal layer of CNS, in particular the subpial granular layer.

a. Low grade glioma – F typically involve WM. However the pleomorphic xanthoastrocytoma (PXA) is cortically-based.
b. DNET - T - benign, focal intracortical mass superimposed on background of cortical dysplasia. Arises from secondary germinal layer of CNS, in particular the subpial granular layer.
c. Pilocytic astrocytoma - F - arises from astrocytic precursor cell. Typically cerebellar mass (hemisphere) which compresses 4th ventricle
d. Cavernous haemangioma - F
e. Ependymoma - F - floor of 4th ventricle mass, slow-growing tumor of ependymal cells.

How well did you know this?
1
Not at all
2
3
4
5
Perfectly
4
Q
  1. Young man with cerebral hemorrhage. Which of the following is less likely:
    a. Amyloidosis
    b. Metastasis
    c. Vasculitis
    d. AVM
    e. Cavernoma
A

a. Amyloidosis AA occurs in patients > 65 years

How well did you know this?
1
Not at all
2
3
4
5
Perfectly
5
Q
  1. Guy having L ICA stent. Develops sudden onset neurological deficits. MRI shows bilateral frontal lobe . Most likely anatomical explanation
    a. azygous ACA
    b. Persistent trigeminal
    c. enlarged posterior communicating artery
A

Answer: Azygous ACA Azygous ACA- single ACA. No AcomA- a/w holoprosencephaly
Persistent trigeminal artery- trigeminal artery that links posterior circulation to ICA - BELOW PcomA level

How well did you know this?
1
Not at all
2
3
4
5
Perfectly
6
Q
  1. Patient presents with acute confusion. MRI shows T2 high signal in the mesiotemporal lobe with restricted diffusion. Most likely diagnosis of
    a. HSV encephalitis
    b. MCA stroke
A

a. HSV encephalitis
B. MCA stroke is wrong- PCA distribution for medial temporal lobe, except for anterior temporal which may be MCA distribution

How well did you know this?
1
Not at all
2
3
4
5
Perfectly
7
Q
Man with gait problems. Fluid density extra dural lesion seen with segmentation abnormalities. Most likely. 
a. epidermoid
b. arachnoid
c neurenteric cyst
d. myelocele
e. nerve sheath tumour
A

Answer
:Neurenteric cyst
Presumably refers to extradural lesion in spinal canal with vertebral segmentation anomalies.

A = epidermoid = F = intramedullary (40%) or extramedullary intradural (60%) (extradural epidermoid rare); follows CSF (except on FLAIR & DWI); assoc/ w/ vertebral abnormalities (hemivertebra, scoliosis); tend to present in early adulthood with slowly progressive symptoms (radiculopathy/myelopathy)

B = arachnoid = F = usually intradural (extradural rare); usually asymptomatic

C = neurenteric cyst = partly intradural & partly extradural; fluid density; assoc/ w/ segmentation anomalies; dumbell lesion; present in 2nd-4th decades with back/radicular pain & gait disturbance

D = myelocele = F

E = nerve sheath tumour = F (not fluid density)

How well did you know this?
1
Not at all
2
3
4
5
Perfectly
8
Q

MRI characteristic features of PML.

a. Diffusion restriction
b. focal white matter abnormalities with sparing of subcortical U fibre
c. typical involvement of middle cerebral peduncle
d. increased aspartate on MRS

A

a. diffusion restriction

Leading edge displays patchy diffusion restriction.

How well did you know this?
1
Not at all
2
3
4
5
Perfectly
9
Q
  1. Regarding myelination of the brain (which is true?):

a. As myelination progresses the white matter becomes T1 hypointense.
b. The posterior internal capsule is myelinated at birth.
c. The optic nerves are myelinated by 2 months of age.
d. The splenium of the corpus callosum myelinates fisrt, myelination then progresses anteriorly.
e. At 24-30/12 the white matter is isointense to grey matter on T2.

A
  • AJL - Multiple are true. (Tell me if you think I’m wrong)
  • Posterior limb of internal capsule is myelinated at birth
  • Optic nerves are myelinated by 2 months of age
  • The corpus callosum myelinates from posterior to anterior starting at splenium.

Previous Answer:d. The splenium of the corpus callosum myelinates first, myelination then progresses anteriorly. dorsal to ventral, caudad to cephalad, central to peripheral. Splenium bright on T1 at 4 mths, genu at 6 mths; then low on T2 by 6 mths and 8 mths respectively. [Kieran’s ‘rules’]

a. As myelination progresses the white matter becomes T1 hypointense. F with myelination, deceased amounts of water → T1 hyperintense & T2 hypointense
b. The posterior internal capsule is myelinated at birth. T? the posterior portion of the posterior limb of IC is myelinated at birth on T1 (2 mo on T2), anterior part of posterior limb by 1mo (4-7 mo on T2); anterior limb at 2-3mo (5-11 mo on T2)
c. The optic nerves are myelinated by 2 months of age. T optic nerves & tracts myelinated by 1 mo
d. The splenium of the corpus callosum myelinates first, myelination then progresses anteriorly. dorsal to ventral, caudad to cephalad, central to peripheral. Splenium bright on T1 at 4 mths, genu at 6 mths; then low on T2 by 6 mths and 8 mths respectively. [Kieran’s ‘rules’]
e. At 24-30/12 the white matter is isointense to grey matter on T2. F

How well did you know this?
1
Not at all
2
3
4
5
Perfectly
10
Q
  1. A 45 year old man presents with a haemorrhage in the right basal ganglia. It demonstrates positive mass effect and is iso- to low intensity to grey matter on both T1 and T2 weighted sequences. The haemorrhage is most likely:
    f. <3hrs old
    g. 8-72 hrs old
    h. Between 1 week and 3 months old
    i. Between 3months and 12 months old.
    j. Over a year old.
A

g. 8-72 hrs old

How well did you know this?
1
Not at all
2
3
4
5
Perfectly
11
Q
  1. Which of the following features is NOT associated with Spontaneous intracranial hypotension?
    a. Subdural hygromas
    b. Diffuse leptomeningeal enhancement
    c. Reduced mamillo-pontine distance
    d. Enlarged pituitary gland
    e. Tonsillar ectopia
A

*AJL b. Diffuse leptomeningeal enhancement F - Pachymeningeal enhancement

  1. Which of the following features is NOT associated with Spontaneous intracranial hypotension? (Osborn II-4-35)
    ANSWER: B

a. Subdural hygromas T
b. Diffuse leptomeningeal enhancement F dural enhancement (pachymeninges); enhancement is smooth, not nodular. Meningeal enhancement in ICH is thick, linear, without nodularity, and involves the pachymeninges of both the infra- and supratentorial compartments without evidence of involvement of the leptomeninges (no abnormal enhancement around the brainstem, within the sylvian fissures, or in the depth of cerebral sulci).
c. Reduced mamillo-pontine distance T
d. Enlarged pituitary gland T pituitary gland enlarged above sella in 50%
e. Tonsillar ectopia T caudal displacement of tonsils 25-75%List- pituitary enlargement- mid brain slump- tonsil ectopia- enlarged dural venous sinus- pachymeningeal enhancement- subdural hygroma/haematoma

How well did you know this?
1
Not at all
2
3
4
5
Perfectly
12
Q
12.	11 yr old female with ataxia and headache has a posterior fossa mass which is hyperdense and enhancing, it is likely to be.
.f.	Medulloblastoma/PNET
g.	Meningioma
h.	Pilocytic astrocytoma
i.	Ependymoma
A
  1. 11 yr old female with ataxia and headache has a posterior fossa mass which is hyperdense and enhancing, it is likely to be.
    ANSWER: A

a. Medulloblastoma/PNET T iso-hyperdense, enhance heterogeneously; seed CSF
b. Meningioma F can be hyperdense & do enhance, but wrong age
c. Pilocytic astrocytoma F partly cystic, only mural nodule enhances
d. Ependymoma F typically hypo-isodense, but commonly have calcification (> 40%); younger age typically

How well did you know this?
1
Not at all
2
3
4
5
Perfectly
13
Q
  1. Mass at the CP angle. Heterogeneously enhancing T1 low and T2 bright, bright on DWI. (Outside – Young Female. T1 iso, T2 hyper, peripheral enhancement. Cystic portions. Restricted diffusion.)
    j. Acoustic schwannoma
    k. Meningioma
    l. Epidermoid
    m. Arachnoid cyst
A
  1. Mass at the CP angle. Heterogeneously enhancing T1 low and T2 bright, bright on DWI. (Outside – Young Female. T1 iso, T2 hyper, peripheral enhancement. Cystic portions. Restricted diffusion.)
    ANSWER: C
    if adding information from bracket

a. Acoustic schwannoma F doesn’t restrict; enhance strongly
b. Meningioma ?F minimal restriction
c. Epidermoid ?T similar to CSF intensity (except on FLAIR), restricted diffusion (this makes the diagnosis); however usually don’t enhance, 25% have mild marginal enhancement
d. Arachnoid cyst F CSF density on all sequence, no restriction

How well did you know this?
1
Not at all
2
3
4
5
Perfectly
14
Q
  1. Which of the following is NOT a feature of Posterior Reversible Encephalopathy syndrome (PRES)?

a. Typically has increased T2 signal in the occipital lobes and cerebellum.
b. May show increased diffusion co-efficient.
c. Posterior circulation vessels are normal.
d. Can uncommonly cause cortical necrosis (ARA some small portions are irreversible)
e. Pathophysiology is the same whether PRES is caused by hypertension or Cyclosporine.

A

c. Posterior circulation vessels are normal.
6. Which of the following is NOT a feature of Posterior Reversible Encephalopathy syndrome (PRES)? Osborn I-10-32Answer: C
a. Typically has increased T2 signal in the occipital lobes and cerebellum (bilateral). T predilection for posterior circulation & involves subcortical WM (but cerebellum not “typical”)
b. May show increased diffusion co-efficient. ?T most common is no restriction, less commonly can have restriction. If they mean ↑ ADC then T – “markedly elevated” ADC values. (T2 shinethrough?)
c. Posterior circulation vessels are normal. ?F At catheter angiography (CA), diffuse vasoconstriction, focal vasoconstriction, vasodilation, and even a string-of-beads appearance have been noted in PRES, consistent with what is typically described as vasospasm or arteritis (AJNR 2008). Osborn says “CTA usually normal. Rare vasospasm with multifocal areas of arterial narrowing”.
d. Can uncommonly cause cortical necrosis (ARA a small portion are irreversible). T rarely causes frank infarctione.
e. Pathophysiology is the same whether PRES is caused by hypertension or Cyclosporin T diverse causes & clinical entities with HTN as common component – acute HTN damages vascular endothelium

How well did you know this?
1
Not at all
2
3
4
5
Perfectly
15
Q
  1. Regarding sellar lesions, which is MOST correct?

a. Dural tail can be a feature of pituitary macroadenoma.
b. Calcification is common in pituitary macroadenoma.
c. Craniopharyngiomas commonly calcify though don’t enhance.
d. Rathke cleft cyst has nodular enhancement.
e. Pituitary macroadenoma demonstrates delayed intense enhancement.

A

ANSWER: Aa. Dural tail can be a feature of pituitary macroadenoma. T subtle/mild dural thickening (“tail”) present in some cases (Osborn II-2-25)
*LW: out of wording of options, this is most correct with STATDx saying same thing re dural tail may be seen.

  1. Regarding sellar lesions, which is MOST correct? Osborn II-2-25

ANSWER: Aa. Dural tail can be a feature of pituitary macroadenoma. T subtle/mild dural thickening (“tail”) present in some cases (Osborn II-2-25) *LW - radprimer agrees.

b. Calcification is common in pituitary macroadenoma. F 1-2% calcify
c. Craniopharyngiomas commonly calcify though don’t enhance. F calcification common in kids (adamant- 90% calcify), but rare in adults; solid portions do enhance heterogeneously
d. Rathke cleft cyst has nodular enhancement. F usually no enhancement
e. Pituitary macroadenoma demonstrates delayed intense enhancement. F most enhance strongly & heterogenously SG thinks this most true *LW - not for MACROadenoma, however MICROadenoma delayed / dynamic enhancement is a feature (Radprimer)

How well did you know this?
1
Not at all
2
3
4
5
Perfectly
16
Q
  1. Drowsy kid with meningitic symptoms. Best diagnostic test.
    a. CSF
    B. CTB noncontrast
    C. CTB post contrast
    d. MRI brain
A

*AJL: Best answer is A - CSF (explanation below)
Starship guidelines on meningitis say: “CT scan should be done if there are any focal neurological signs. Herniation may occur even in the presence of a normal scan. A normal head CT scan does not exclude presence of raised intracranial pressure and should not influence your decision to perform a lumbar puncture; this is a clinical decision.”
With regards to low GCS they say: “Lumbar puncture Contra-indicated in presence of coma (GCS <9), raised intra-cranial pressure or unstable clinical state. If meningitis is suspected but LP is contra-indicated, start antibiotics”

Drowsy is not a focal neurological sign. Also, even if they were super drowsy (eg <9) there is no mention of doing head imaging.
Therefore they seem to recommend do LP and don’t worry about imaging. (I note SG, presumably Sam, agrees)

**LJS agree, especially since it says “best diagnostic test”. CTB is not diagnostic, just excludes SOL prior to LP. Would depend on wording - if it were “what is the next test” and kid had significantly lowered GCS I would chose CT

***: LW: general consenus agrees :)

  1. Drowsy kid with meningitic symptoms. Best diagnostic test.
    a. CSF
    B. CTB noncontrast
    C. CTB post contrast
    d. MRI brain

Previous answers….
ANS = B. Assuming reduced GCS, CTB probably indicated before LP (see below)
SG – if just drowsy (GCS 14 ish) - CSF

French guidelines: CNS imaging is indicated if:
• Focal neurology
• GCS ≤ 11
• Seizures if > 5, only if hemiseizure in kids < 5

RCH guidelines on LP & CT:
CT Scans if focal neurological signs
• CT Scans are not helpful in most children with meningitis.
• A normal CT scan does not tell you that the patient does not have raised ICP
.• Herniation may occur even in the presence of a normal scan.
• Don’t delay antibiotics whilst waiting for a CT

UK NICE guidelines:
Use clinical assessment and not cranial computed tomography (CT), to decide whether it is safe to perform a lumbar puncture.
CT is unreliable for identifying raised intracranial pressure.
In children and young people with a reduced or fluctuating level of consciousness (Glasgow Coma Scale score less than 9 or a drop of 3 or more) or with focal neurological signs, perform a CT scan to detect alternative intracranial pathology.

How well did you know this?
1
Not at all
2
3
4
5
Perfectly
17
Q

Post partum .Head ache.

Bilateral thalamic abnormality. Most likely.

A

venous thrombosis of internal cerebral vein of Galen

How well did you know this?
1
Not at all
2
3
4
5
Perfectly
18
Q

young patient post neck manipulation. Ataxia, ptosis, nystagmus and some other eye stuff. MRA. Most likely to see

a. dissection of carotids
b. dissection of V3 portion of vertebral artery

A

ANS = B ⇒ symptoms suggest posterior circulation

ICA dissection- C1 (more common then VA), 2cm above bifurcation- C2- C5
Vertebral- V2 or V3

How well did you know this?
1
Not at all
2
3
4
5
Perfectly
19
Q

In patient with dissection, least likely to see

a. intracranial haematoma
b. wall haematoma
c. narrowed true lumen

A

a. intracranial haematoma least likely. - infarct more likely+/- haemorrhage transformation
Wall haematoma best seen with FAT SAT

How well did you know this?
1
Not at all
2
3
4
5
Perfectly
20
Q

18 yo. Mass in 3rd ventricular region. Contains calc and enhancing avidly. Negative bHCG in CSF. Most likel
a. germinoma
B. met

A

A = germinoma (engulfs calcifications of pineal gland, enhance intensely)

How well did you know this?
1
Not at all
2
3
4
5
Perfectly
21
Q

Patient with mesotemporal sclerosis. Most likely finding.

a. contralateral maxillary enlargement
b. ipsilateral hippocampal atrophy
c. ipsilateral maxillary atrophy

A

b. ipsilateral hippocampal atrophy
Signs of mesial temporal sclerosis (Osborn I-10-62)
• Abnormal T2 hyperintensity
• Hippocampal volume loss/atrophy
• Obscuration of internal architecture, loss of grey-white differentiation
• Hippocampus > amygdala > fornix > mamillary bodies

How well did you know this?
1
Not at all
2
3
4
5
Perfectly
22
Q

AUGUST 201022) Previous extracranial ICA stent, subsequent parietal and superior cerebellar infarct: (Note previous exam question was Occipital)

i) Persistent trigeminal
ii) Fetal posterior circulation
iii) Persistent hypoglossal
iv) ICA aneurysm
v) Azygous artery

A

ANSWER:i) Persistent trigeminal T (see below, Saltzmann type 2) the most common ICA-BA anomalous connection

1) Previous extracranial ICA stent, subsequent parietal and superior cerebellar infarct: (Note previous exam question was Occipital)
i) Persistent trigeminal T (see below, Saltzmann type 2) the most common ICA-BA anomalous connection
ii) Fetal posterior circulation F above SCA; would be correct if occipital & parietal lobe affected
iii) Persistent hypoglossal T also possible, as joins basilar artery low down, but rare (parallels CN12 in hypoglossal canal)
iv) ICA aneurysm F
v) Azygous artery F (single/fused ACA from A2 distal)

Trigeminal bridges precavernous ICA to the basilar artery below the level of the PCOM. The basilar artery below the anastamosis can be hypoplastic and enlarged above. The PCOM can also be small.

Saltzman type 1: PTA supplies distal BA, pcomms absent, prox BA hypoplastic

Saltzman type 2: PTA feed superior cerebellar arteries, patent pcomms suppy the PCA’s. Hypoglossal artery arises from the extracranial ICA at C2-3 level, traverses the hypoglossal canal and anastamoses with the basilar artery low down.

How well did you know this?
1
Not at all
2
3
4
5
Perfectly
23
Q

23) Regarding holoprosencephaly, which is least correct:
i) Is associated with endocrine abnormalities
ii) Increase in incidence likely secondary to more awareness of mild form
iii) Caused by mutations in Sonic Hedgehog gene
iv) Lobar form associated with non fusion of central grey matter

A

ii) Increase in incidence likely secondary to more awareness of mild forms ??F
2) Regarding holoprosencephaly, which is least correct:
i) Is associated with endocrine abnormalities (T - J Pediatr Endocrinol Metab. 2005 Oct;18(10):935-41)
ii) Increase in incidence likely secondary to more awareness of mild forms ??F
iii) Caused by mutations in Sonic Hedgehog gene (T – Robbins/Osborn) Sonic hedgehog SHH (7q36), SIX3 (2p21), TGIF1 (18p11.3)
iv) Lobar form associated with non fusion of central grey matter (T - Unlike semilobar holoprosencephaly, the falx is present, the interhemispheric fissure is fully formed and the thalami are not fused)

How well did you know this?
1
Not at all
2
3
4
5
Perfectly
24
Q

24) 16yo male. Wedge shaped area of decreased CT attenuation involving both cortex and white matter of posterior temporal lobe. No contrast enhancement. Most likely:
1) DNET
ii) Embolism
iii) Ganglioglioma

A

ANSWER: i) DNET T can mimic stroke on CT. Cortically-based mass. Clasically wedge-shaped & “bubbly” at MRI.

3) 16yo male. Wedge shaped area of decreased CT attenuation involving both cortex and white matter of posterior temporal lobe. No contrast enhancement. Most likely:
ANSWER: i) DNET T can mimic stroke on CT. Cortically-based mass. Clasically wedge-shaped & “bubbly” at MRI.
ii) Embolism
iii) Ganglioglioma Most common cause of temporal lobe epilepsy (Osborn).

How well did you know this?
1
Not at all
2
3
4
5
Perfectly
25
Q

25) Dilated VR spaces. Which of the following is false
i) MPS
ii) Metachromatic leukodystrophy
iii) Cysticercosis
iv) Craniopharyngioma
v) Muscular dystrophies

A

**LJS - I think metachromatic leukodystrophy is false. The tigroid pattern is due to sparing of normal signal perivascular WM within high T2 demyelinated WM. I think this is what they are testing.

Paper on leukodystrophies: https://pubs.rsna.org/doi/full/10.1148/radiographics.22.3.g02ma01461

Paper on perivascular spaces: https://pubs.rsna.org/doi/full/10.1148/rg.274065722

Previous ANSWER: probably V (muscular dystrophies - but seen in myotonic dystrophy)

4) Dilated VR spaces. Which of the following is false
i) MPS T (Osborn)
ii) Metachromatic leukodystrophy T (not in Osborn, but found elsewhere)
iii) Cysticercosis T
iv) Craniopharyngioma T Edema-like change along the optic tract commonly occurs in association with craniopharyngiomas… and in association with pituitary region tumors other than craniopharyngiomas. It is related to distension of normally present large Virchow-Robin spaces adjacent to the optic tract. Because Virchow-Robin spaces are speculated to be a drainage route of interstitial fluid, their distension may be related to the fluid retention in and along the Virchow-Robin spaces, the outlet of which into the subarachnoid space is blocked by pituitary region tumors. [AJNR 2003 24: 336-342

]v) Muscular dystrophies F/T seen in myotonic dystrophy

Dilated Virchow-Robin spaces / basal ganglia cystic lesions
•	Normal variant
•	Vascular
o	Lacunar infarcts
o	Amyloid angiopathy
•	Infectious
o	Cryptococcosis (AIDS; look for miliary enhancing nodules!)
o	Neurocysticercosis
o	Tuberculosis
•	Congenital/idiopathic
o	Mucopolysaccharidoses (Hunter, Hurler – esp. CC & peritrigonal WM)
o	Sarcoidosis
o	Sturge-Weber syndrome
o	Meningioangiomatosis
o	Meningiomelanomatosis
o	Metachromatic leukodystrophy
Types- type I: along lenticulostriate arteria entering basal ganglia - type II: along the path of perforating medullary arteries as they enter the cortical GM over the high convexities - type III: in the midbrain. [RG 2007]
How well did you know this?
1
Not at all
2
3
4
5
Perfectly
26
Q

26) 30 year old woman with cerebellar mass and ataxia. On CT the mass is hypodense with an enhancing nodule. The most likely cause is
i) Pilocytic astrocytoma
ii) Medulloblastoma
iii) Dysembryoplastic neuroepithelial tumour
iv) Haemangioblastoma
v) Metastasis

A

iv) Haemangioblastoma

How well did you know this?
1
Not at all
2
3
4
5
Perfectly
27
Q

27) 24yo male. Previously well. Presents with dysarthria and CN VII, XI, XII palsies. MRI shows restricted diffusion in premotor cortex. Most likely cause:
i) Seizure
ii) Migraine related vasospasm
iii) Paradoxical emboli
iv) MCA thrombus

A

ANSWER:iii) Paradoxical emboli T isolated cortical infarction may result from distal embolic infarcts (ClinRad 2008)

6) 24yo male. Previously well. Presents with dysarthria and CN VII, XI, XII palsies. MRI shows restricted diffusion in premotor cortex. Most likely cause:
i) Seizure ?F changes usually more diffuse, typically cortex &/or subcortical WM (Osborn – Status epilepticus topic).
ii) Migraine related vasospasm
iii) Paradoxical emboli T isolated cortical infarction may result from distal embolic infarcts (ClinRad 2008)
iv) MCA thrombus- 7, 11 and 12 are all involved in the corticobulbar tract, this suggests a pseudobulbar palsy- The restricted diffusion in the premotor cortex is likely an infarct as we have no history of headaches or seizures in the question.- A first time seizure in this region is a possibility but likely would be secondary suggesting a paradoxical emboli as the most likely.- The MCA thrombus would involve a larger area??

How well did you know this?
1
Not at all
2
3
4
5
Perfectly
28
Q

28) Consultant asks you what is most likely intracranial cyst to calcify. You reply
:i) Colloid cyst
ii) Choroid plexus cyst
iii) Arachnoid cyst

A

ANSWER:ii) Choroid plexus cyst T irregular, peripheral calcification in majority

7) Consultant asks you what is most likely intracranial cyst to calcify. You reply:

1) Colloid cyst F rarely calcify
ii) Choroid plexus cyst T irregular, peripheral calcification in majority
iii) Arachnoid cyst F not mentioned in Osborn

How well did you know this?
1
Not at all
2
3
4
5
Perfectly
29
Q

29) Sellar region. Which is most correct:
i) 17mm is within normal height for a post partum females pituitary
ii) Papillary craniopharyngiomas are more commonly cystic then adenomatous (their spelling, not mine) cranios
iii) A suprasellar arachnoid cyst displaces the pit stalk anteriorly
iv) Absence of sellar bony enlargement suggests meningioma over adenoma

A

ANSWER:iv) Absence of sellar bony enlargement suggests meningioma over adenoma T large adenomas expand sellar & may erode its floor, while meningioma is usually separated from sella by diaphragma sellae (probably the most correct answer, although…)

8) Sellar region. Which is most correct:
i) 17mm is within normal height for a post partum females pituitary F ≤ 12mm
ii) Papillary craniopharyngiomas are more commonly cystic then adenomatous (their spelling, not mine) cranios F papillary form more commonly solid than adamantinomatous form
iii) A suprasellar arachnoid cyst displaces the pit stalk anteriorly T 15% of arachnoid cysts arise in the suprasellar region, and they can have local mass effect (Req p371)
iv) Absence of sellar bony enlargement suggests meningioma over adenoma T large adenomas expand sellar & may erode its floor, while meningioma is usually separated from sella by diaphragma sellae (probably the most correct answer, although…)

How well did you know this?
1
Not at all
2
3
4
5
Perfectly
30
Q

30) Old dude with known coronary artery disease. Ovoid mass with areas of calcification adjacent to left sphenoid sinus. Most likely:
i) Fusiform basilar artery aneurysm
ii) Fusiform ICA aneurysm
iii) CoW berry aneurysm
iv) Micotic MCA aneurysm

A

ii) Fusiform ICA aneurysm T

How well did you know this?
1
Not at all
2
3
4
5
Perfectly
31
Q

31) Cyst in area of 4th ventricle. Least likely:
i) Porencephalic cyst
ii) Arachnoid cyst
iii) Villous hypertrophy
iv) Subependymal cyst

A

ANSWER:iii) Villous hypertrophy F hypertrophy of villi of choroid plexus. Osborn Radiology 2006: “Villous hyperplasia is very rare and, when present, enhances strongly and relatively uniformly.” AJR 2009 (Naeini): “Diffuse villous hyperplasia is a rare condition that usually involves the lateral ventricles and is manifested by bilateral enlargement of the entire choroid plexus without discrete masses, resulting in overproduction of CSF and communicating hydrocephalus”.

10) Cyst in area of 4th ventricle. Least likely:

i) Porencephalic cyst F usually adjacent to lateral ventricle, typically supratentorial
* *“LJS - can get in cerebellum

ii) Arachnoid cyst F atypical location
iii) Villous hypertrophy F hypertrophy of villi of choroid plexus. Osborn Radiology 2006: “Villous hyperplasia is very rare and, when present, enhances strongly and relatively uniformly.” AJR 2009 (Naeini): “Diffuse villous hyperplasia is a rare condition that usually involves the lateral ventricles and is manifested by bilateral enlargement of the entire choroid plexus without discrete masses, resulting in overproduction of CSF and communicating hydrocephalus”.
iv) Subependymal cyst F rare in 4th ventricle

How well did you know this?
1
Not at all
2
3
4
5
Perfectly
32
Q

MARCH 201032) A 5 year old presents with unsteady gait and lethargy. The neurosurgeon wants to know if a posterior fossa lesion may be a pilocytic astrocytoma. This would be suggested by the imaging appearance of a :

a. Hypodense mass in the cerebellum.
b. Solid hyperdense mass in the midline cerebellum with enhancement.
c. Solid hypodense mass in the midline cerebellum with no enhancement
d. Hypodense mass in a cerebellar hemisphere with no enhancement.
e. Hypodense mass in a cerebellar hemisphere with a nodule that enhances

A

e. Hypodense mass in a cerebellar hemisphere with a nodule that enhances

How well did you know this?
1
Not at all
2
3
4
5
Perfectly
33
Q

33) A 5 year old presents with unsteady gait and lethargy. The neurosurgeon wants to know if this lesion may be a medulloblastoma. This would be suggested by the imaging appearance of a :
a. Hypodense mass in the cerebellum.
b. Solid hyperdense mass in the midline cerebellum with enhancement.
c. Solid hypodense mass in the midline cerebellum with no enhancement
d. Hypodense mass in a cerebellar hemisphere with no enhancement.
e. Hypodense mass in a cerebellar hemisphere with a nodule that enhances.

A

b. Solid hyperdense mass in the midline cerebellum with enhancement.

How well did you know this?
1
Not at all
2
3
4
5
Perfectly
34
Q

34) 43. 70yo with TIA, US shows extracranial ICA stenosis 70-99%. As of January 2010, the current recommended treatment is:
a. Randomised controlled trial has shown medical therapy better than endarterectomy.
b. RCT shown endarterectomy better than medical
c. RCT shown stenting better than endarterectomy
d. RCT shown endarterctomy better than stenting
e. No randomised controlled trials have been performed to say which treatment is better.

A

ANSWER: B; RCT shown endarterectomy better than medical T (i.e. NASCET, ECST)

  1. 70yo with TIA, US shows extracranial ICA stenosis 70-99%. As of January 2010, the current recommended treatment is:
    a. Randomised controlled trial has shown medical therapy better than endarterectomy. F at this degree of stenosis there is RCT proof that surgery is better
    b. RCT shown endarterectomy better than medical T (i.e. NASCET, ECST)
    c. RCT shown stenting better than endarterectomy F The results of randomized trials have not shown consistent outcome differences between CAS and CEA. CAS may be superior to CEA in certain patient groups, such as those exposed to previous neck surgery or radiation injury.
    d. RCT shown endarterectomy better than stenting T MJA 2010: Recent evidence from RCTs has cast doubt on the safety of widespread use of stenting for the treatment of patients with symptomatic or asymptomatic carotid stenosis. The overall results from these RCTs indicate that carotid endarterectomy (endarterectomy) is still the preferred treatment option for symptomatic carotid stenosis. PAH was in one of the trials for CAS, but it finished eary due to the above.
    e. No randomised controlled trials have been performed to say which treatment is better.
How well did you know this?
1
Not at all
2
3
4
5
Perfectly
35
Q

35) 27. 30 female left neck pain, 10 hrs of diplopia and dysarthria. Normal non contast CT head and neck. The next investigation should be:
a. Lumbar puncture
b. Contrast enhanced CT head
c. DSA
d. Carotid US
e. MRI brain

A

ANSWER:e. MRI brain T if with T1 fat sat neck and MRA neck/brain, although according to RG 08 has quite poor sens/spec for vertebral artery dissection. MRI brain to exclude other causes other than dissection

  1. 30 female left neck pain, 10 hrs of diplopia and dysarthria. Normal non contast CT head and neck. The next investigation should be:

a. Lumbar puncture F
b. Contrast enhanced CT F, but would be T if CTA head/neck
c. DSA ?T invasive, but “gold standard”
d. Carotid US F
e. MRI brain T if with T1 fat sat neck and MRA neck/brain, although according to RG 08 has quite poor sens/spec for vertebral artery dissection. MRI brain to exclude other causes other than dissection

Obsorn – best imaging tool (vertebral dissection)
• CTA of neck with coronal + sagittal reconstructions usually diagnostic
• MR including axial T1WI with fat-sat + TOF MRA will make the diagnosis in most cases; Gd MRA better
• In equivocal cases or in cases where endovascular treatment is contemplated (SAH, hemodynamically significant stenosis) catheter angiography should be performed

How well did you know this?
1
Not at all
2
3
4
5
Perfectly
36
Q
36.	Woman on OCP. Bilateral haemorrhagic thalamic and BG infarcts.
A.	Superior sagital sinus thrombosis 
B.	Inferior sagital sinus 
C.	Vein of Labbe
D.	Internal cerebral vein 
E.	Cavernous sinus
A

ANSWER:D. Internal cerebral vein T (pink) – would have to involve both veins; a better answer would be vein of Galen (great cerebral vein) or straight sinus
*LW: Agree with wording, singular word of vein cats some doubt, although likely best available option.

  1. Woman on OCP. Bilateral haemorrhagic thalamic and BG infarcts.

A. Superior sagital sinus thrombosis F (green)
B. Inferior sagital sinus F
C. Vein of Labbe F temporal lobe (blue)
D. Internal cerebral vein T (pink) – would have to involve both veins; a better answer would be vein of Galen (great cerebral vein) or straight sinus
E. Cavernous sinus F

How well did you know this?
1
Not at all
2
3
4
5
Perfectly
37
Q
37.	AVM (which is true?)
A.	CT with contrast pick up >90% 
B.	V of Galen is AVM 
C.	Venous angioma haemorrhage 
D.	Cavernous haemangioma seen on angiogram 
E.	Typically have mass effect
A

ANSWER:A. CT with contrast pick up >90% T (“strong enhancement of arterial feeders, nidus & draining veins”), can’t find figures

  1. AVM (Osborn I-5-4)

A. CT with contrast pick up >90% T (“strong enhancement of arterial feeders, nidus & draining veins”), can’t find figures
B. V of Galen is AVM F Osborn – AVF between deep choroidal arteries & large midline varix (embryonic median prosencephalic vein of Markowski)
C. Venous angioma haemorrhage F venous angioma = DVA = do not normally bleed (if bleed look for co-existent cavernoma). Capillary telangiectasias also do not bleed unless other associated malformation (e.g. cavernoma) present.
D. Cavernous haemangioma seen on angiogram F (cavernoma angiographically occult)
E. Typically have mass effect F RG 2010 “mass effect is a rare pathomechanism that may result from large venous ectasias or compression of critical structures by the nidus”. StatDx “minimal or no mass effect”

How well did you know this?
1
Not at all
2
3
4
5
Perfectly
38
Q
  1. 35 yr old with cerebellar hemisphere mass. Hyperdense on CT. Enhance.
    f. Meningioma
    g. Pilocytic astrocytoma
    h. Medulloblastoma
    i. Ganglioglioma
    j. DNET
A

ANSWER: c. Medulloblastoma T? hemispheric MB’s can occur in adults, almost always < 40 years, but still most occur in children < 20 years

  1. 35 yr old with cerebellar hemisphere mass. Hyperdense on CT. Enhance.

a. Meningioma F? extra-axial (if didn’t specify in the cerebellar hemisphere this would probably be best answer, although usually in older age group)
b. Pilocytic astrocytoma F cyst + nodule, childhood
c. Medulloblastoma T? hemispheric MB’s can occur in adults, almost always < 40 years, but still most occur in children < 20 years
d. Ganglioglioma F like peripheral temporal lobes, rare in cerebellum
e. DNET F peripheral, supratentorial

How well did you know this?
1
Not at all
2
3
4
5
Perfectly
39
Q
  1. 30 year old woman presents with history of ataxia and headache. Contrast CT shows cystic lesion in cerebellar hemisphere with enhancing nodule. Which is likely?
    k. Ganglioglioma
    l. Pilocytic astrocytoma
    m. Haemangioblastoma
A

c. Haemangioblastoma T in this age; cyst + nodule

How well did you know this?
1
Not at all
2
3
4
5
Perfectly
40
Q
  1. Patient having left extracranial ICA angiogram (stent). Sudden neurological symptoms. MRI shows left parietal and occipital infarct. Most likely…
    a. Persistent trigeminal
    b. Fetal posterior circulation
    c. Persistent hypoglossal
    d. ICA aneurysm
A

*AJL - I favour persistent trigeminal (if patient had fetal right circulation this would explain the unilateral changes).
Fetal left circulation with emboli could explain it as well but they seem to be angling towards posterior circulation changes which fits best with peristent trigeminal.

**LJS could be either but PComA more common than trigeminal

**LW:
Favor Fetal PCA:
–> commonly shows anterior and posterior distribution, with posterior distribution being PCA territory i.e. occipital lobe rather than cerebellar involvement, so if the ICA is trashed during procedure down stream effects will thus be seen in anterior and PCA territory.

Persistent trigeminal artery (PTA), from intracavernous ICA to BA, so normally have a degree of cerebellar perfusion, and in this case would show MRI features in the left cerebellum.

Previous answer:
a. is not true because PCA and ACA will be affected- parieto-occipital infarct- superior cerebellar ifnarct

b. fetal PCA is more correct because only PCA distribution affected

  1. Patient having left extracranial ICA angiogram (stent). Sudden neurological symptoms. MRI shows left parietal and occipital infarct. Most likely…
    a. Persistent trigeminal
    b. Fetal posterior circulation
    c. Persistent hypoglossal
    d. ICA aneurysm
How well did you know this?
1
Not at all
2
3
4
5
Perfectly
41
Q
41.	70yr old. MRI for dementia. Cyst in the atria of left lateral ventricle.
A.	Choroid plexus cyst 
B.	Ependymal cyst 
C.	Mets 
D.	Choroids plexus papilloma
A

ANSWER : AA. Choroid plexus cyst T most common choroid plexus mass in adults; common in elderly & fetuses, up to 40% in elderly, often bilateral. Common location is at the atria. (Osborn I-7-38)

  1. 70yr old. MRI for dementia. Cyst in the atria of left lateral ventricle.ANSWER : A

A. Choroid plexus cyst T most common choroid plexus mass in adults; common in elderly & fetuses, up to 40% in elderly, often bilateral. Common location is at the atria. (Osborn I-7-38)
B. Ependymal cyst F rare and usu younger (Osborn I-7-42)
C. Mets F can cause intraventricular mass, less common c.f. CPC
D. Choroids plexus papilloma F in kids in lateral ventricles; adults get in 4th ventricle

How well did you know this?
1
Not at all
2
3
4
5
Perfectly
42
Q
  1. Gelastic seizure and precocious puberty:

n. Tuber cinerum hamartoma

A

a. Tuber cinerum hamartoma T non-enhancing hypothalamic mass contiguous with tuber cinereum. Located between pons/mammillary bodies and hypothalamic infundibulumAnatomy of the hypothalamus

• Forms the floor (& part of the walls) of the 3rd ventricle & is located inferior to the thalamus
• Made up of (from anterior to posterior):
o Optic chiasm
o Tuber cinereum
– a convex sheet of grey matter extending posterosuperiorly from the infundibulum
– located b/w the optic chiasm & mamillary bodies

o Infundibular stalk
– leading down to posterior lobe of pituitary gland

o Mamillary bodies
– small round masses anterior to the posterior perforated substance in which the columns of the fornix (vide infra) end

How well did you know this?
1
Not at all
2
3
4
5
Perfectly
43
Q
  1. Adrenoleukodystrophy, which is least correct:
    a More common in males
    b. There is enhancement
    c. Asymmetrical involvement
    d. Involves periatrial and splenium of the corpus callosum
A

ANSWER: C. normally symmetrical

  1. Adrenoleukodystrophy, which is least correct: (SK) (Osborn I-9-40)’ANSWER:

a. More common in males T X-linked condition – severe progressive form usually affects pre-teen males
b. There is enhancement T leading edge enhancing; enhancement strongly linked to progression
c. Asymmetrical involvement F usually symmetric
d. Involves periatrial and splenium of the corpus callosum T
e. Affects the white matter, adrenals, and testicles T affects Leydig cells in testes

Note
- x linked disease
- metabolic disease
- mechanism: lack of myelination of veery long change fatty asid
- peritrogonal posterior pattern
- symmetrica
- enhance
- around atrium and selenium of corpus callosum
• Enhancing peritrigonal demyelination; leading edge enhances
• Location
o Peritrigonal white matter (WM)
o Pattern: Splenium → peritrigonal WM → corticospinal tracts/fornix/commisural fibers/visual and auditory pathways
o Typically spares subcortical U-fibers
• Morphology
o Usually symmetrical, confluent, posterior involvement; rare frontal pattern occurs
o Central (splenium) to peripheral gradient is usual

How well did you know this?
1
Not at all
2
3
4
5
Perfectly
44
Q
44.	Dilated perivascular spaces. False
A.	Can have mass effect 
B.	May confuse with lacunar infarct 
C.	Often involves the cortex 
D.	Suppress on FLAIR
A

ANSWER: C
C. Often involves the cortex F (BG, mid brain, deep white matter, extreme capsule)
*AJL - while I agree with the answer above, dilated perivascular space can involve the cortex though I’m guessing not ‘often’ (as per radiopaedia there are 3 types; basal ganglia, cortical grey matter, midbrain)

  1. Dilated perivascular spaces. False (SK) (Osborn I-7-30)

A. Can have mass effect T focal mass effect can occur (giant/tumefactive PVS)
B. May confuse with lacunar infarct T
C. Often involves the cortex F (BG, mid brain, deep white matter, extreme capsule)
D. Suppress on FLAIR T suppress completely, although 25% have minimal increased signal in brain surrounding enlarged PVS.

How well did you know this?
1
Not at all
2
3
4
5
Perfectly
45
Q
  1. A cerebral infarct can be differentiated from penumbra by:
    a. Decreased cerebral flow (CBF)
    b. Increased cerebral flow (CBF)
    c. Decreased cerebral volume (CBV)
    d. Increased cerebral volume (CBV)
    e. Increased mean transit time (MTT)
    f. Decreased MTT
A

c. Decreased cerebral volume (CBV)

How well did you know this?
1
Not at all
2
3
4
5
Perfectly
46
Q
46.	Craniopharyngioma.  True:
A.	Typical age 30 
B.	More likely to be intrasellar 
C.	Can have nodular/curvilinear calcification 
D.	MRI: most likely show solid mass
A

ANSWER: C
C. Can have nodular/curvilinear calcification T 90% of adamantinatous forms calcify, but rarely calcify in papillary type. Adamantinomatous mixed solid-cystic, while papillary type often solid.

  1. Craniopharyngioma. True (SK) (Osborn II-2-33)

A. Typical age 30 F 5-15 (kids, adamantinomatous) and >50 for papillary squamous

B. More likely to be intrasellar F supra 75%, both 20% and intra 4%

C. Can have nodular/curvilinear calcification T 90% of adamantinatous forms calcify, but rarely calcify in papillary type. Adamantinomatous mixed solid-cystic, while papillary type often solid.

D. MRI: most likely show solid mass F in kids (adamantinomatous) solid & cystic, while papillary type often solid.

How well did you know this?
1
Not at all
2
3
4
5
Perfectly
47
Q
  1. MBA, confused 9/12 later. Best sequence for DAI?
    a. 3D spoiled gradient
    b. DWI
    c. FLAIR
    d. T2* GRE
    e. 3D GRE
    f. FSE
A

ANSWER : 3D GRE = SWI

  1. MBA, confused 9/12 later. Best sequence for DAI?
    a. 3D spoiled gradient F These sequences allow for fast 3D imaging during short apnea (10 to 20 seconds). They are used in angiography after gadolinium injection
    b. DWI
    c. FLAIR
    d. T2* GRE good, but not as good as SWI
    e. 3D GRE (i.e. SWI)
    f. FSE

Susceptibility-weighted imaging (SWI) is a 3D, flow-compensated, radiofrequency spoiled gradient-recalled echo sequence that takes advantage of susceptibility variations between tissues. SWI combines magnitude and phase images to detect these differences. (AJR Oct 2010; JMIRO 2010).

How well did you know this?
1
Not at all
2
3
4
5
Perfectly
48
Q
  1. Cerebral vascular lesions, which is true?
  2. 90% are detected on CT
  3. Angiography detects cavernomata
  4. Vein of Galen aneurysm is an AVM
  5. Venous angiomas bleed more commonly in posterior fossa than anteriorly
  6. AVMs are most commonly dural-based
A

ANSWER: A
90% are detected on CT (?? T)

  1. Cerebral vascular lesions, which is true?
    a. 90% are detected on CT (?? T)
    b. Angiography detects cavernomata F cavernoma angiographically occult
    c. Vein of Galen aneurysm is an AVM F? Osborn – AVF between deep choroidal arteries & large midline varix (embryonic median prosencephalic vein of Markowski)
    d. Venous angiomas bleed more commonly in posterior fossa than anteriorly F venous angioma = DVA = do not normally bleed (if bleed look for co-existent cavernoma)
    e. AVMs are most commonly dural-based F most common type is a parenchymal (pial) AVM
How well did you know this?
1
Not at all
2
3
4
5
Perfectly
49
Q
  1. Regarding MRI T1 weighted imaging of the brain, which is true?
    a. Best sequence for imaging haemorrhage (subarachnoid blood)
    b. Calcium is bright (hyperintense)
    c. Grey matter is hyperintense compared to white matter
    d. Posterior pituitary is of lower signal intensity than gray matter
    e. Signal within the superior sagittal sinus is a reliable indicator of thrombosis
A

ANSWER: b. Calcium is bright (hyperintense) T/F certain crystalline forms of calcium are T1 hyper. Intracranial calcifications show variable MRI signal characteristics and have an unspecific appearance, making them difficult to characterize. MRI cannot reliably rule out or determine the presence of calcifications. (Radiologia 2006)

  1. Regarding MRI T1 weighted imaging of the brain, which is true?
    a. Best sequence for imaging haemorrhage (subarachnoid blood) F for SAH use FLAIR, for other blood use T2* or SWI; T1 good for subacute blood
    b. Calcium is bright (hyperintense) T/F certain crystalline forms of calcium are T1 hyper. Intracranial calcifications show variable MRI signal characteristics and have an unspecific appearance, making them difficult to characterize. MRI cannot reliably rule out or determine the presence of calcifications. (Radiologia 2006)
    c. Grey matter is hyperintense compared to white matter F - at T1 GM is hypointense, WM is hyperintense
    d. Posterior pituitary is of lower signal intensity than gray matter F bright spot
    e. Signal within the superior sagittal sinus is a reliable indicator of thrombosis F signal intensity is variable, and acute venous thrombus can mimic a normal flow state (RG 2006)
How well did you know this?
1
Not at all
2
3
4
5
Perfectly
50
Q

AUGUST 200950. NOT associated with vHL?

a. Cerebellar hemangioblastoma
b. Renal cell carcinoma
c. Phaeochromocytoma
d. Endolymphatic sac tumors
e. Mucinous adenoma of the pancreas

A

ANSWER:e. Mucinous adenoma of the pancreas F

  1. NOT associated with vHL?

a. Cerebellar hemangioblastoma – 40-70%, most frequent cause of death
b. Renal cell carcinoma –50-66% of patients.
c. Phaeochromocytoma – 10-15%
d. Endolymphatic sac tumors <10-15% of VHL
e. Mucinous adenoma of the pancreas F

How well did you know this?
1
Not at all
2
3
4
5
Perfectly
51
Q
  1. Which of the following is NOT a characteristic CT feature of acute herpes simplex encephalitis
    a. Sparing of the basal ganglia
    b. Unilateral changes seen on MRI
    c. Evidence of hemorrhage on MRI
    d. Involvement of the limbic system
    e. Mass effect
A

b. Unilateral changes seen on MRI F - bilateral but asymmetrical

HSV-1 causes 95% of all herpetic encephalitis. HSV-2 more common in neonates. Haemorrhagic, necrotizing encephalitis of gray and white matter (primarly limbic system). May se mild patchy enhancement. DWI - may see restricted diffusion. Spares the basal ganglia, bilateral but asymmetrical.

How well did you know this?
1
Not at all
2
3
4
5
Perfectly
52
Q
  1. 21yo woman with Hx of headaches and ‘pressure symptoms’ for years. Large cystic mass in right temporal lobe with calcified mural nodule and scalloping of inner table of skull. MOST LIKELY?
    a. Pilocytic astrocytoma
    b. Lhermette-Duclos disease
    c. Ganglioglioma
    d. Low grade glioma
    e. Oligodendroglioma
A

ANSWER:c. Ganglioglioma T most likely; most commonly cyst + mural nodule, produce TLE (>75% in temporal lobe), often calcified

  1. 21yo woman with Hx of headaches and ‘pressure symptoms’ for years. Large cystic mass in right temporal lobe with calcified mural nodule and scalloping of inner table of skull. MOST LIKELY? Osborn I-6-82
    a. Pilocytic astrocytoma F uncommon location (cerebellum>optic nerve/chiasm>adjacent to 3rd ventricle>brainstem)
    b. Lhermette-Duclos disease F in cerebellum; striated cerebellar hamartoma; Cowden disease
    c. Ganglioglioma T most likely; most commonly cyst + mural nodule, produce TLE (>75% in temporal lobe), often calcified
    d. Low grade glioma F commonest tumour, but unlikely to have cyst+nodule
    e. Oligodendroglioma – more solid, more calcium, slightly older
Cyst + nodule neoplasm DDx
•	Posterior fossa
o	Pilocytic astrocytoma 
o	Haemangioblastoma 
•	Supratentoria
lo	Craniopharyngioma
o	Ganglioglioma (& DIG in infants < 1 yr)
o	Pleomorphic xanthoastrocytoma (PXA)
o	S-PNET (uncommon variant)
o	Schwannoma (intraparenchymal, rare)
How well did you know this?
1
Not at all
2
3
4
5
Perfectly
53
Q
  1. 30yo female. Abnormal gait and headache of a few weeks duration. No previous history of note. NeuroSx want to exclude a post fossa tumor. Which (CT) would suggest medulloblastoma as MOST LIKELY diagnosis.

a. Hyperdense, contrast enhancing mass arising from cerebellar vermia
b. Hyperdense, contrast enhancing mass involving the left cerebellar hemisphere
c. Hyperdense, non contrast enhancing mass involving the cerebellar vermis
d. Hyper dense, unilateral enhancing mass involving the brain stem
e. Hyperdense mass with an enhancing nodule involving the left cerebellar hemisphere.

A

ANSWER:b. Hyperdense, contrast enhancing mass involving the left cerebellar hemisphere T 50-89% arise laterally (cerebellar hemisphere) in older children & adults (Osborn)

  1. 30yo female. Abnormal gait and headache of a few weeks duration. No previous history of note. NeuroSx want to exclude a post fossa tumor. Which (CT) would suggest medulloblastoma as MOST LIKELY diagnosis.
    a. Hyperdense, contrast enhancing mass arising from cerebellar vermia (>90 % hyperdense and > 90% enhance on CT) in children arises from roof of 4th ventricle, i.e. midline lesion
    b. Hyperdense, contrast enhancing mass involving the left cerebellar hemisphere T 50-89% arise laterally (cerebellar hemisphere) in older children & adults (Osborn)
    c. Hyperdense, non contrast enhancing mass involving the cerebellar vermis
    d. Hyper dense, unilateral enhancing mass involving the brain stem
    e. Hyperdense mass with an enhancing nodule involving the left cerebellar hemisphere.
How well did you know this?
1
Not at all
2
3
4
5
Perfectly
54
Q
  1. Regarding cerebral venous thrombosis, which is FALSE? (they decided to repeat this question x2 in the exam)

a. Most common cause in adults in infection
b. Higher risk during puerperium than in pregnancy
c. In adults, a coagulopathy is the most important contributing factor
d. May be mimicked by subdural hematoma on unenhanced CT
e. May be accompanied by subarachnoid on CT

A

ANSWER: A
. Most common cause in adults in infection F – Infections contribute to < 10% of cerebral venous thrombosis (40% in children though). Resistance to activated protein C (typically due to factor V Leiden mutation) = most common cause of sporadic CVT. (StatDx)

  1. Regarding cerebral venous thrombosis, which is FALSE? (they decided to repeat this question x2 in the exam) (SK/RS)
    a. Most common cause in adults in infection F – Infections contribute to < 10% of cerebral venous thrombosis (40% in children though). Resistance to activated protein C (typically due to factor V Leiden mutation) = most common cause of sporadic CVT. (StatDx)
    b. Higher risk during puerperium than in pregnancy T Majority of studies have reported a higher proportion of CVT during puerperium compared to pregnancy
    c. In adults, a coagulopathy is the most important contributing factor T
    d. May be mimicked by subdural hematoma on unenhanced CT T layered blood on tentorium cerebelli may mimic transverse sinus thrombosis (Osborn)
    e. May be accompanied by subarachnoid on CT T Am J Emerg Med Jan 2009.
How well did you know this?
1
Not at all
2
3
4
5
Perfectly
55
Q
  1. The following regarding DAI of brain is TRUE?

a. The dorsal brain stem is commonly affected
b. Most lesions are haemorrhagic
c. Genu is the portion of corpus callosum most often involved
d. Acute lesions are of low signal on T2W MRI
e. Lesions characteristically involve the cortex

A

ANSWER:a. The dorsal brain stem is commonly affected F? only in severe cases, poor prognosis. Neuro Req “lesions are also seen in the dorsolateral quadrant of the rostral (superior) part of the brainstem, adjacent to the superior cerebellar peduncle. Brainstem DAI associated with more profound injury.”

  1. The following regarding DAI of brain is TRUE? (SK)
    a. The dorsal brain stem is commonly affected F? only in severe cases, poor prognosis. Neuro Req “lesions are also seen in the dorsolateral quadrant of the rostral (superior) part of the brainstem, adjacent to the superior cerebellar peduncle. Brainstem DAI associated with more profound injury.”
    b. Most lesions are haemorrhagic F 80% of lesions are microscopic, non-haemorrhagic (Osborn I-2-36)
    c. Genu is the portion of corpus callosum most often involved F splenium and under surface of posterior body
    d. Acute lesions are of low signal on T2W MRI F Neuro Req “high intensity on T2/FLAIR +/- haemorrhage”; Osborn - depends if haemorrhagic or non-haemorrhagic
    e. Lesions characteristically involve the cortex F grey-white junction

o Characteristic locations:
occur at interfaces b/w tissues
• Cerebral hemispheric grey-white matter junctions (67%)
– milder injuries confined to parasagittal frontal lobes & periventricular temporal lobes
• Corpus callosum (esp. posterior body & splenium)
– in 20%
• Dorsolateral brainstem & superior cerebellar peduncles
– in severe cases
• Basal ganglia & internal capsule

How well did you know this?
1
Not at all
2
3
4
5
Perfectly
56
Q
  1. Regarding imaging in the Immunocompromised patient, which is TRUE?
    a. Low density lymphadenopathy on CT is characteristic of Kaposi’s sarcoma
    b. MRI is sensitive to early changes in HIV encephalitis
    c. Pneumocystic carinii pneumonia typically involves the lung bases following Rx with inhaled pentamidine
    d. MAC infection, peripheral lymphadenopathy is usually more marked than abdominal lymphadenopathy
    e. Toxoplasmosis lesions occur most commonly in the basal ganglia
A

ANSWER:e. Toxoplasmosis lesions occur most commonly in the basal ganglia T( BG are a favored site as well as corticomedullary junction, thalamus & cerebellum) – Osborn I-8-86. Also consider lymphoma, esp. if solitary.

  1. Regarding imaging in the Immunocompromised patient, which is TRUE?

a. Low density lymphadenopathy on CT is characteristic of Kaposi’s sarcoma F – hyperenhancing in 2/3.
b. MRI is sensitive to early changes in HIV F
c. Pneumocystic carinii pneumonia typically involves the lung bases following Rx with inhaled pentamidine F upper lobes
d. MAC infection, peripheral lymphadenopathy is usually more marked than abdominal lymphadenopathy F
e. Toxoplasmosis lesions occur most commonly in the basal ganglia T( BG are a favored site as well as corticomedullary junction, thalamus & cerebellum) – Osborn I-8-86. Also consider lymphoma, esp. if solitary.

How well did you know this?
1
Not at all
2
3
4
5
Perfectly
57
Q
  1. Not seen in NF1
    a. Rib notching
    b. Clinodactyly
    c. Dural ectasia
    d. Scoliosis
    e. Lateral thoracic meningocele
A

b. Clinodactyly F T21 or familial clinodactyly

How well did you know this?
1
Not at all
2
3
4
5
Perfectly
58
Q
  1. Which of the following conditions is NOT associated with agenesis or hypogenesis of the corpus callosum
    a. Interhemispheric lipoma
    b. Chiari I malformation
    c. Absence of the anterior commisure
    d. Gray matter heteropia
    e. Sphenoidal menigo-encephalocele
A

b. Chiari I malformation

How well did you know this?
1
Not at all
2
3
4
5
Perfectly
59
Q
  1. Regarding T1 imaging of brain, which is TRUE?
    a. Posterior pituitary is of lower signal intensity than gray matter
    b. Calcification is a recognised cause of hyperintense signal
    c. Signal within the superior sagittal sinus is a reliable indicator of thrombosis
    d. Gray matter is higher signal than white matter
    e. Is optimal sequence for indentifying subarachnoid blood
A

ANSWER:b. Calcification is a recognised cause of hyperintense signal T certain forms of crystalline calcium are T1 hyperintense

  1. Regarding T1 imaging of brain, which is TRUE?
    a. Posterior pituitary is of lower signal intensity than gray matter F T1 bright spot.
    b. Calcification is a recognised cause of hyperintense signal T certain forms of crystalline calcium are T1 hyperintense
    c. Signal within the superior sagittal sinus is a reliable indicator of thrombosis F artefactual ↑ signal common
    d. Gray matter is higher signal than white matter F vice versa
    e. Is optimal sequence for indentifying subarachnoid blood F FLAIR best
How well did you know this?
1
Not at all
2
3
4
5
Perfectly
60
Q
  1. Regarding hemangioblastomas of the CNS, which is FALSE?
    a. Most common primary infra-tentorial intra-axial tumour in adults
    b. Majority are confined to paravermian cerebellum
    c. Associated with secondary polycythemia
    d. Associated with syringomyelia
    e. Calcification is common
A

ANSWER:e. Calcification is common F calcification very rare (not described in Osborn)

  1. Regarding hemangioblastomas of the CNS, which is FALSE? Osborn I-6-143

a. Most common primary infra-tentorial intra-axial tumour in adults T
b. Majority are confined to paravermian cerebellum T? 80% in cerebellar hemispheres
c. Associated with secondary polycythemia T secrete EPO
d. Associated with syringomyelia T? presumably
e. Calcification is common F calcification very rare (not described in Osborn)

How well did you know this?
1
Not at all
2
3
4
5
Perfectly
61
Q
  1. Which is NOT a feature of idiopathic intracranial hypertension
    a. Cranial nerve palsy
    b. Orthostatic headache, worse when upright
    c. Transverse sinus stenosis
    d. Cerebral venous thrombosis
    e. Clinical improvement with placement of intrasinus stents
A

ANSWER:b. Orthostatic headache, worse when upright F feature of intracranial hypotension (Osborn II-4-34). BIH/IIH presents in obese woman with headache & papilloedema. Headache in BIH exacerbated by Valsalva.

  1. Which is NOT a feature of idiopathic intracranial hypertension (BIH) Osborn I-10-40

a. Cranial nerve palsy T CN 6 most commonly. Progressive visual loss, diplopia.
b. Orthostatic headache, worse when upright F feature of intracranial hypotension (Osborn II-4-34). BIH/IIH presents in obese woman with headache & papilloedema. Headache in BIH exacerbated by Valsalva.
c. Transverse sinus stenosis T cause or effect – debated
d. Cerebral venous thrombosis T? must exclude this diagnosis
e. Clinical improvement with placement of intrasinus stents T

How well did you know this?
1
Not at all
2
3
4
5
Perfectly
62
Q
  1. Which is FALSE regarding idiopathic intracranial hypertension?
    a. Cranial neve palsies
    b. Orthostatic postural headache
    c. Caused by sinus stenosis
    d. Caused by sinus thrombosis
    e. Symptoms improve with sinus shunting
A

b. Orthostatic postural headache

How well did you know this?
1
Not at all
2
3
4
5
Perfectly
63
Q

MARCH 2009 EXAM RECALLS

  1. A bilateral infarct involving the basal ganglia and thalami could be best explained by thrombosis of:
  2. Superior sagittal sinus
  3. Inferior sagittal sinus
  4. Veins of Labbe
  5. Internal cerebral veins 5.Cavernous sinus
A

4.Internal cerebral veins - T - The deep system includes the vein of Galen, internal cerebral veins, and their tributaries; the Rosenthal vein (basal vein) and its tributaries; and the medullary and subependymal veins, which drain the hemispheric white matter. The deep system drains the inferior frontal lobe; most of the deep WM of the frontal, temporal, and parietal lobes; corpus callosum; upper brainstem; basal ganglia; and thalamus. The parenchymal alterations that occur with deep venous occlusion typically involve the thalami, probably because the primary venous pathways that drain the thalami extend directly into the internal cerebral veins (see diagram).

How well did you know this?
1
Not at all
2
3
4
5
Perfectly
64
Q
  1. Dilated VR spaces. Which of the following is false?
  2. MPS
  3. Metachromatic leukodystrophy
  4. Cysticercosis
  5. Craniopharyngioma
  6. Muscular dystrophy
A

*LW…per previous LJS answer and logic, metachromatic leukodystrophy is favoured to be false being a mimic

ANSWER: 5
5.Muscular dystrophy - F - however, in myotonic dystrophy (amplification of an unstable CTG trinucleotide repeat sequence in a protein kinase gene on chromosome 19), the cerebral MRI features include diffuse atrophy (68%), subcortical WM lesions (65%), wide VR spaces (38%) and thickening of the skull (35%). [Neuroradiology 1996]

2.Dilated VR spaces. Which of the following is false? (GC)

  1. MPS - T - inherited disorder of metabolism characterised by enzyme deficiency and inability to break down glycosaminoglycan (GAG). VR spaces are dilated by accumulated GAG, esp. peritrigonal region.
  2. Metachromatic leukodystrophy - T - may have a ‘tigroid’ or ‘leopard’ apprearance due to perivascular sparing.
  3. Cysticercosis - T - fluid-filled ovoid cyst +/- scolex
  4. Craniopharyngioma - T - Because VR spaces are speculated to be a drainage route of interstitial fluid into the subarachnoid space, their distension may be related to the fluid retention in and along the VR spaces, the outlet of which into the subpial and/or subarachnoid space(s) is blocked by pituitary region tumors. [AJNR 2003]
  5. Muscular dystrophy - F - however, in myotonic dystrophy (amplification of an unstable CTG trinucleotide repeat sequence in a protein kinase gene on chromosome 19), the cerebral MRI features include diffuse atrophy (68%), subcortical WM lesions (65%), wide VR spaces (38%) and thickening of the skull (35%). [Neuroradiology 1996]
How well did you know this?
1
Not at all
2
3
4
5
Perfectly
65
Q
  1. Regarding myelination in children, which is of the following is the most correct?
  2. Myelination causes reduced / low signal on T1
  3. The anterior internal capsule is myelinated at birth.
  4. White matter and grey matter appear the same on T2 at 24-30 months
  5. Optic nerves myelinated by two months.
  6. From the splenium of the corpus callosum forwards
A

ANSWER:5.From the splenium of the corpus callosum forwards - T - dorsal to ventral, caudad to cephalad, central to peripheral. Splenium bright on T1 at 4 mths, genu at 6 mths; then low on T2 by 6 mths and 8 mths respectively. [Kieran’s ‘rules’]

3.Regarding myelination in children, which is of the following is the most correct? (GC)

  1. Myelination causes reduced / low signal on T1 - F - bright on T1 (fatty myelin).
  2. The anterior internal capsule is myelinated at birth - F - at birth, there is myelination of dorsal WM tracts of brainstem, cerebellar peduncles, dorsomedial tracts of diaencephalon, posterior limb of IC, and WM in the post-central gyrus. The anterior limb and genu of CC myelinate 4 mths later that the posterior limb & splenium. [Clinical MRI 2006]
  3. White matter and grey matter appear the same on T2 at 24-30 months - F - adult pattern attained by 18 mths.
  4. Optic nerves myelinated by two months – T? - assume this means 2 months after birth… The optic nerves and tracts are myelinated by one month, optic radiations at 3 months. [Osborn]
  5. From the splenium of the corpus callosum forwards - T - dorsal to ventral, caudad to cephalad, central to peripheral. Splenium bright on T1 at 4 mths, genu at 6 mths; then low on T2 by 6 mths and 8 mths respectively. [Kieran’s ‘rules’]
How well did you know this?
1
Not at all
2
3
4
5
Perfectly
66
Q
  1. 60 year old woman on a tread mill. Brain haemorrhage in posterior frontal lobe and corona radiata. MRI and angiogram normal other than the area of bleed. DSA normal. Becomes well again. MRI three months later shows old haematoma as the only abnormality. The cause of the haemorrhage is best explained by:
  2. Hypertensive bleed
  3. Amyloid angiopathy
  4. Malignancy
  5. Bleed into a metastasis
  6. AVM
A

ANSWER:1.Hypertensive bleed - T - atypical location (usually BG, thalamus, pons, cerebellum). However, more likely in a patient of this age, and that the repeat MRI shows old haematoma only. Also accounts for history/exercise, which means she’d be unlikely to be dementing from her amyloid angiopathy.

4.60 year old woman on a tread mill. Brain haemorrhage in posterior frontal lobe and corona radiata. MRI and angiogram normal other than the area of bleed. DSA normal. Becomes well again. MRI three months later shows old haematoma as the only abnormality. The cause of the haemorrhage is best explained by: (GC, TW, JS)

ANSWER:
1.Hypertensive bleed - T - as above

  1. Amyloid angiopathy - F – should see other foci of small haemorrhae on T2* GRE. According to the Boston criteria, this patient would fit into “possible” CAA category (clinical data suggesting CAA and the imaging finding of a single cortical-subcortical hematoma in a patient older than 55yrs, without other cause of hemorrhage). [RG 2006]
  2. Malignancy - F - should see on imaging.
  3. Bleed into a metastasis - F - should see on imaging. Correlate with history of malignancy.
  4. AVM - F - Parietal >frontal >temporal. MRI: flow voids, TOF. Angio: “bag of worms”.
How well did you know this?
1
Not at all
2
3
4
5
Perfectly
67
Q
  1. 10 year old child is drowsy with a headache. The most appropriate next investigation is:
  2. Observation
  3. CT brain
  4. CT brain with contrast
  5. LP
  6. MRI
A

*LW:
per previous meningitic question stem logic, it would depend on degree on drowsyness and if menigitis is being questioned clinically (which it should based on this stem), LP is preferred investigation if ?meningitis.

3.CT brain non contrast
Indications for neuroimaging in children with headache: High priority:
acute headache, worst headache of life, thunderclap headache chronic-progressive pattern, focal neuro symptoms, abnormal neuro examination, papilloedema, abnormal eye movements, hemiparesis, ataxia, abnormal reflexes, VP shunt, neurocutaneous syndrome, age <3 years

Moderate priority: headaches or vomiting on awakening, unvarying location of headache, meningeal signs. [Guideline for diagnosis and management of migraine headaches in chlidren, Gunner et al 2008]

How well did you know this?
1
Not at all
2
3
4
5
Perfectly
68
Q

6.A young female has neck pain, dysarthria and diplopia. Non contrast CT brain and cervical spine both normal. The next most appropriate investigation is:
1.CT brain with contrast
2.MRI brain
3.Duplex ultrasound neck
4 DSA
5. LP

A
  1. MRI brain - T - MR is the modality of choice (but should specify MRI + MRA?). MRI detects both the intramural thrombus and intimal flap that are characteristic of VAD. Hyperintensity of the vessel wall seen on T1 fat sat axial images is considered pathognomonic of VAD. MRA can identify abnormalities that are characteristic of the disturbed arterial flow seen in VAD. These include the presence of a pseudolumen and aneurysmal dilation of the artery. MRI and MRA are less sensitive than cerebral angiography for the detection of VAD, although they probably have equivalent specificity.
  2. A young female has neck pain, dysarthria and diplopia. Non contrast CT brain and cervical spine both normal. The next most appropriate investigation is: (GC)
  3. CT brain with contrast - F - possibly CTA (accessibility after hours).
  4. MRI brain - T - MR is the modality of choice (but should specify MRI + MRA?). MRI detects both the intramural thrombus and intimal flap that are characteristic of VAD. Hyperintensity of the vessel wall seen on T1 fat sat axial images is considered pathognomonic of VAD. MRA can identify abnormalities that are characteristic of the disturbed arterial flow seen in VAD. These include the presence of a pseudolumen and aneurysmal dilation of the artery. MRI and MRA are less sensitive than cerebral angiography for the detection of VAD, although they probably have equivalent specificity.
  5. Duplex ultrasound neck - F - demonstrates abnormal flow in 95% of patients with VAD; US signs specific to VAD (eg, segmental dilation of the vessel, eccentric channel) are detectable in only 20% of patients.
  6. DSA - T - indicated when clinical suspicion is high but MRI/MRA has failed to isolate the lesion; characteristic angiographic finding in a dissected vertebral artery is the string or “string and pearl” appearance of the stenotic vessel lumen; also intimal flap / complete occlusion.
  7. LP - F - patients with suspected SAH and a normal CT scan may undergo LP if VAD is not pursued by other imaging modalities. The typical presentation of VAD is a young person with severe occipital headache and posterior nuchal pain following a recent, relatively minor, head or neck injury. The trauma is generally from a trivial mechanism but is associated with some degree of cervical distortion.

Focal neurologic signs attributable to ischaemia of the brainstem / cerebellum ultimately develop in 85% of pts; however, a latent period as long as 3 days between onset of pain and development of CNS sx is not uncommon.

Symptoms of vertebral a. dissection include: Ipsilateral facial dysaesthesia (pain and numbness) - most common symptom Dysarthria or hoarseness (CN IX and X) Contralateral loss of pain and temperature sensation in the trunk and limbs
Ipsilateral loss of taste (nucleus and tractus solitarius) Hiccups Vertigo Nausea and vomiting Diplopia or oscillopsia (image movement experienced with head motion) Dysphagia (CN IX and X) Disequilibrium Unilateral hearing loss [eMedicine; Craniocervical arterial dissection RG 2008]

How well did you know this?
1
Not at all
2
3
4
5
Perfectly
69
Q
  1. An elderly man has an MRI that shows a lesion which is isointense on T1, hyperintense on T2 and some enhancement post gadolinium. (Also recalled as isointense on T1 and hypointense on T2??) The most likely explanation is
  2. Haemorrhage into a cavernoma
  3. Haemorrhage into a metastasis
  4. Amyloid angiopathy
  5. Hypertensive haemorrhage
A

ANSWER:2.Haemorrhage into a metastasis - T - may see enhancement if hemorrhage is secondary to neoplasm (hyperacute blood Iso/Hyper, or Acute blood Iso/Hypo). Although peripheral “ring” enhancement of a hematoma (vascularized capsule) is possible after a few days this is in subacute phase.

  1. An elderly man has an MRI that shows a lesion which is isointense on T1, hyperintense on T2 and some enhancement post gadolinium. (Also recalled as isointense on T1 and hypointense on T2??) The most likely explanation is (TW)
  2. Haemorrhage into a cavernoma - F - cavernoma: minimal or no enhancement (unless mixed with other lesion eg DVA).
  3. Haemorrhage into a metastasis - T - may see enhancement if hemorrhage is secondary to neoplasm (hyperacute blood Iso/Hyper, or Acute blood Iso/Hypo). Although peripheral “ring” enhancement of a hematoma (vascularized capsule) is possible after a few days this is in subacute phase.
  4. Amyloid angiopathy - F
  5. Hypertensive haemorrhage - F - shouldn’t show enhancement acutely, see ans 1.
How well did you know this?
1
Not at all
2
3
4
5
Perfectly
70
Q
  1. 30 year old woman with cerebellar mass and ataxia. On CT the mass is hypodense with an enhancing nodule. The most likely cause is
  2. Pilocytic astrocytoma
  3. Medulloblastoma
  4. Dysembryoplastic neuroepithelial tumour
  5. Haemangioblastoma
  6. Metastasis
A

4.Haemangioblastoma - T - familial VHL associated HGBL occur at younger age, sporadiac HGBL peak is 40-60yo. Intraaxial posterior fossa mass with cyst, enhancing mural nodule, abutting pia.

How well did you know this?
1
Not at all
2
3
4
5
Perfectly
71
Q
  1. Which of the following would be most likely demonstrated on an MRI of a cerebral abscess?
  2. A smooth inner margin
  3. No rim enhancement
  4. Increased signal on the ADC
  5. Increased lactate
  6. Increased choline
A

ANSWER:1.A smooth inner margin - T - as the ring-enhancing lesion of a cerebral abscess is classically described as having a smooth inner margin and smooth outer margin (abscess wall appears thickened on the gray matter or “oxygen” side, and thinner along the white matter or ventricular side. Occasionally the outer perimeter of the abscess can appear irregular).

  1. Which of the following would be most likely demonstrated on an MRI of a cerebral abscess? (TW, GC)
  2. A smooth inner margin - T - as the ring-enhancing lesion of a cerebral abscess is classically described as having a smooth inner margin and smooth outer margin (abscess wall appears thickened on the gray matter or “oxygen” side, and thinner along the white matter or ventricular side. Occasionally the outer perimeter of the abscess can appear irregular).SK – Neuro Req p199 – ring is smoother on outside than inside, but Primer says opposite! Lin says smooth margin suggests abscess over a nodular/irregular margin of neoplasm, RG article on enhancement agrees. Osborn doesn’t go into this.
  3. No rim enhancement - F - thin, distinct enhancing rim (early capsule); thickened enhancement capsule (late capsule).
  4. Increased signal on the ADC - F - restricted diffusion (reduced ADC, increased DWI)
  5. Increased lactate – T - but less specific as central necrotic area of tumour may show lactate
  6. Increased choline - F - Typical MRS: absent signals of NAA, creatine, and choline.
72
Q
  1. CT perfusion angiogram of an infarct. Which of the following would be observed regarding the penumbra, best answer combination (1-5 below)?
    a. Increased blood flow
    b. Decreased blood flow
    c. Increased blood volume
    d. Decreased blood volume
    e. Increased mean transit time
  2. a, c, e
  3. b, e
  4. b, d, e
  5. b, c, e
  6. a, e
A

4.b, c, e
Pneumbra shows either:

(a) increased MTT with moderately decreased CBF (>60%) and normal or increased CBV (80-100% or higher) secondary to autoregulatory mechanisms; or
b) increased MTT with markedly reduced CBF (>30%) and moderately reduced CBV (>60%). Infarcted tissue shows severely decreased CBF (<30%) and moderately reduced CBV (<40%) with increased MTT.

When the capillary bed dilates in the early stage of ischemia, CBV will remain unchanged or even increase despite a decrease in CBF. When mechanisms of autoregulation can no longer compensate for a further reduction in CBF, CBV will also decrease.

73
Q
  1. Multiple dilated VR spaces are seen in all of the following except, least correct answer
  2. Cryptococcus
  3. Craniopharyngioma
  4. Mucopolysaccharidoses
  5. Microvascular disease
  6. Increased age
A

ANSWER :2.Craniopharyngioma – T? - oedema like change occurs in association with pituitary region tumors in the adjacent brain parenchyma along the optic tract - related to distension of VR spaces. More focal changes. Least correct.

11.Multiple dilated VR spaces are seen in all of the following except, least correct answer (TW)

  1. Cryptococcus - T - opportunistic fungal infection. CNS infection can be either meningeal or parenchymal (usually starts as meningitis, most pronounced at base of brain). As infection spreads along VR spaces they may become distended with mucoid.
  2. Craniopharyngioma – T? - oedema like change occurs in association with pituitary region tumors in the adjacent brain parenchyma along the optic tract - related to distension of VR spaces. More focal changes. Least correct.
  3. Mucopolysaccharioses - T - inherited disorder of metabolism characterised by enzyme deficiency and inability to break down glycosaminoglycan (GAG). VR spaces are dilated by accumulated GAG.
  4. Microvascular disease - T - arteriosclerotic microvascular disease. Clinically dilated VR spaces are associated with aging, dementia, incidental WM lesions, and hypertension and other vascular risk factors.
  5. Increased age - T - with advancing age VR spaces are found with increasing frequency and larger apparent size (>2mm).
74
Q
  1. An astrocytoma can be best differentiated from an ependymoma by which of the following?
  2. Well-defined intramedullary lesion
  3. Poorly defined intramedullary lesion
  4. Haemorrhage
  5. Homogeneous enhancement
  6. Multiple meningiomas
A

ANSWER:2.Poorly defined intramedullary lesion - T - astrocytoma is poorly defined, homogeneous extensive ill-defined cord tumor with expansion of the cord.

  1. An astrocytoma can be best differentiated from an ependymoma by which of the following? (TW) Lin p310
  2. Well-defined intramedullary lesion - F - ependymoma has well defined margins on contrast enhanced images (but Dahnert also says well-demarcated/diffusely infiltrating cord tumor).
  3. Poorly defined intramedullary lesion - T - astrocytoma is poorly defined, homogeneous extensive ill-defined cord tumor with expansion of the cord.
  4. Haemorrhage - F - hemorrhage uncommon in astrocytoma, more in ependymoma.
  5. Homogeneous enhancement - F - astrocytomas have patchy irregular Gd enhancement; ependymomas mostly have intense homogeneous enhancement on Gd MR.
  6. Multiple meningiomas - F - NF2 - multiple inherited schwannomas, meningiomas, ependymomas.
75
Q
  1. Which of the following is the least specific regarding atlanto-occipital instability?
  2. Occipital condylar fracture
  3. > 12mm basion – dens distance
  4. > 12mm distance from basion to posterior axial line
  5. Extradural haemorrhage along the anterior aspect of the spinal canal
  6. Flexion / extension views and survival of the fittest!
A

**LJS - disagree. Flexion/extension views are for atlanto-axial instability, this question is asking about atlanto-occipital instability. There is not much osseous stability at the atlanto-occipital joint and stability relies on ligaments (esp alar lig and tectorial membrane). So I would say extradural haemorrhage in the location of these ligaments would be pretty concerning for ligamentous injury. I also think atlanto-occpital instability is pretty much always due to high energy trauma, seems unlikely to be doing dynamic manoeuvres!

Good paper about craniocervical junction injuries:
https://pubs.rsna.org/doi/10.1148/rg.2015150035

*LW: as always, agrees with LJS….
Important to ensure question stems refers to traumatic A-O instability, as then if traumatic, XR is relatively pointless in the age of CT and MRI….
However, is non traumatic AO instability, flexion and extension radiographs are still used by some spinal surgeons, and in the non traumatic setting, extradural blood would be unlikely, so would alter the answer
(i think this is highly unlikely, and the stem would be traumatic based).

*AJL - I disagree. Although the flexion/extension views are not typically performed, IF they were performed they could be very specific for instability.
However, haemorrhage in the anterior aspect of the spinal canal does not tell you about spinal stability therefore is non-specific.
To put it another way, if one saw haemorrhage in the anterior spinal canal without another finding, one would not put a diagnosis of atlanto-axial instability in the conclusion.

Previous ANSWER: 4.Extradural haemorrhage along the anterior aspect of the spinal canal - non specific.

  1. Which of the following is the least specific regarding atlanto-occipital instability? (GC)
  2. Occipital condylar fracture - T - classified as impaction fractures, extensions of occipital skull fractures or avulsion fractures at the insertion of the alar ligaments. The latter are potentially unstable fractures, esp. if displaced, and when assocd with tectorial memebrane injury can result in gross atlanto-occipital discontinuity. May be unilateral or bilateral, may extend in a ringlike pattern around the foramen magnum, and are extemely difficult to identify on xray. May have a lower CN palsy (most commonly CNXII due to fracture extension into hypoglossal canal).
  3. > 12mm basion-dens distance - T
  4. > 12mm distance from basion to posterior axial line - T - anteriorly, see below.
  5. Extradural haemorrhage along the anterior aspect of the spinal canal - non specific.
  6. Flexion / extension views and survival of the fittest! - T
76
Q
  1. 40 year old female presents with symptoms of subarachnoid haemorrhage and a 1cm bilobed aneurysm of the anterior communicating artery. Which of the following is the most correct?
  2. Rest and reduce blood pressure
  3. Surgical clipping
  4. Treatment with calcium antagonist therapy
  5. DSA and waiting
  6. IV mannitol
A

ANSWER:3.Treatment with calcium antagonist therapy - T - prophylaxis with oral nimodipine: CCBs have been shown to reduce the incidence of ischaemic neurological deficits, and nimodipine has been shown to improve overall outcome within 3mths of aneurysmal SAH. Although the mechanism is unproved, it appears that nimodipine may prevent the ischaemic complications of vasospasm by the neuroprotective effect of blockading the influx of calcium into damaged neurons. CCBs and other antihypertensives should be used cautiously to avoid the deleterious effects of hypotension.

14.40 year old female presents with symptoms of subarachnoid haemorrhage and a 1cm bilobed aneurysm of the anterior communicating artery. Which of the following is the most correct? (GC)

  1. Rest and reduce blood pressure - F
  2. Surgical clipping - F - could coil or clip in this case.
  3. Treatment with calcium antagonist therapy - T - prophylaxis with oral nimodipine: CCBs have been shown to reduce the incidence of ischaemic neurological deficits, and nimodipine has been shown to improve overall outcome within 3mths of aneurysmal SAH. Although the mechanism is unproved, it appears that nimodipine may prevent the ischaemic complications of vasospasm by the neuroprotective effect of blockading the influx of calcium into damaged neurons. CCBs and other antihypertensives should be used cautiously to avoid the deleterious effects of hypotension.
  4. DSA and waiting - F - ruptured aneurysms should be treated urgently (within 72 h of hemorrhage) to prevent rebleeding and to permit aggressive management of vasospasm.
  5. IV mannitol - F - this patient ‘presents’, implies she is not high grade. Mannitol is used for those with signs of increased ICP or herniation as it decreases the ICP by 50% after 30min. These pts should be intubated and hyperventilated (pCO2 30-45mmHg; excessive hyperventilation may potentiate vasospasm).
77
Q
  1. Hypoxic injury in premature neonate
  2. periventricular watershed
  3. external watershed
  4. peri rolandic infarction
A

1.periventricular watershed T - vascular watershed (“border zone”) shifts from periventricular in premature to parasagittal in term infant at about 36wks.

Also remember that before 26wks there is no gliosis, vs after 26 wks where develop PVL (vs porencephaly).

< 28 weeks - hydranencephaly ,
porencephaly28 - 32
- germinal matrix haemorrhage 32 - 36
- PVL New Born-> cortex (perirolandex cortex), deep grey matter ( basal ganglia, thalamus). or-> prolonged partial asphyxia cause watershed distribution

78
Q
  1. Contraindications MRI (which is false?)
  2. PPM
  3. clipped aneurysm
  4. Stent
  5. Spinal stimulator
  6. Spinal rods
A

*AJL - contraindications have changed since this question was written. pretty much all the things listed are no longer contraindications assuming they are modern enough.
Probably the closest contraindication is clipped aneurism because often the clip type is not easily identified if the operation was not a recent one.

ANSWER:5.Spinal rods - F - heating is relatively low. Generally orthopedic implants show no deflection within main magnetic field.

  1. Contraindications MRI (TW)
  2. PPM - T - absolute contraindication for MRI. Even in patients where PM has been removed, remaining pacer wires could act as an antenna and induce currents, causing cardiac fibrillation.
  3. clipped aneurysm - T - the presence of some intracranial aneurysm clips is an absolute contra-indication to MRI. Clip motion may damage the vessel. Only MRI if definitely know clip is non-ferrous.
  4. Stent - T - less false - depending on what stent. Coronary artery stents are largely MRI safe from what I can gather, as are aortic stents. Not sure about intracranial stents, or stents elsewhere - didn’t specify in answer.
  5. Spinal stimulator - T - certain implated devices are contraindicated for MR inaging becuase they are either, magnetically, electrically, or mechanically activated (cochlear implants, tissue expanders, ocular prosthesis, dental implants, neurostimulators, bone growth timulators, implantable cardiac defibrillators, implantable drug infusion pumps)
  6. Spinal rods - F - heating is relatively low. Generally orthopedic implants show no deflection within main magnetic field.
79
Q
  1. What is not a contraindication to contrast
  2. seafood anaphylaxis
  3. Multiple myeloma
  4. reduced GFR
  5. previous reaction to contrast
  6. severe thyrotoxicosis
A

*LW:
Based on below logic, the only actual contra indications are previous severe (i assume anaphylaxis) reaction to contrast, and severe thyrotoxicosis.

Contra indications based Per RANZCR and CDHB guidelines….
1.seafood anaphylaxis: Not a contra indication –> Shellfish allergy is not associated with an increased risk of adverse reaction to intravenous iodinated contrast media, over and above the approximate 3-fold increased risk associated with other food allergies.

2.Multiple myeloma: cant find this mentioned in most recent guidelines, so dont think it is currently a contra indication.

3.reduced GFR: not really a contraindication –> eGFR ≤29 is not an absolute contraindication to iodinated contrast administration. In this situation, the referring clinician decides whether the scan is required or whether an alternative imagine modality could answer the clinical question.
–> further RANZCR recommendation: Intravascular iodinated contrast media should be given to any patient regardless of renal function status if the perceived diagnostic benefit to the patient, in the opinion of the
radiologist and the referrer, justifies this administration.

  1. previous reaction to contrast: TRUE contra indication.
  2. severe thyrotoxicosis: relatively TRUE contra indication –> Acute thyrotoxicosis/hyperthyroid – avoid unless life threatening.

ANSWER:2.Multiple myeloma – T? – listed in RANZCR guidelines as “risk factor for contrast-induced nephropathy”

  1. What is not a contraindication to contrast (TW/SK)
  2. seafood anaphylaxis - ?F - severe allergy (misconception that patients allergic to fish or shellfish are at additional increaed risk for adverse reactions to contrast, beyond that of any atopic individual or patients with other food allergies. Likely explanation is that seafood is a common cause for food allergy, and individuals with any atopic condition are at higher risk for contrast reactions). RANZCR lists “history of allergy requiring medical treatment”
  3. Multiple myeloma – T? – listed in RANZCR guidelines as “risk factor for contrast-induced nephropathy”
  4. reduced GFR - F - listed in RANZCR guidelines as “risk factor for contrast-induced nephropathy”. Risk of precipitating ARF
  5. previous reaction to contrast - F - increased risk. RANZCR lists.
  6. severe thyrotoxicosis - F - medical conditions which are associated with adverse reactions to iodinated contrast (hyperthyroid, homozygous sickle, myasthenia gravis [but low risk]). Listed by RANZCR.
80
Q

AUGUST 200818.Which has not been shown to cause reversible diffusion restriction?

  1. Seizures
  2. DAI
  3. Spongiform encephalopathies
  4. PRES
A

ANSWER:3.Spongiform encephalopathies - F (?best answer) - high signal on DWI thought to represent vacuolization of he neutrophil. In CJD DWI hyperintensity may disappear later in disease and may be attributable to changes in brain tissue from mild spongioform degeneration to status spongiosus (not ‘reversible per se’).

  • *LJS - ? PRES does not typically have diffusion restriction.
  • *LW: DWI: usually normal, sometimes hyperintense due to oedema (T2 shine-through) or true restricted diffusion
  1. Which has not been shown to cause reversible diffusion restriction? (TW)
  2. Seizures - T - post seizures can get areas of restricted diffusion from cell oedema
  3. DAI - T - most DAI lesions are microscopic nonhaemorrhagic lesions - inertial injury and axons are stretched (not sheared - except in severe injury) - get surrounding swelling / oedema and ‘auto-axotomy’. Oedema - reversible diffusion
  4. Spongiform encephalopathies - F (?best answer) - high signal on DWI thought to represent vacuolization of he neutrophil. In CJD DWI hyperintensity may disappear later in disease and may be attributable to changes in brain tissue from mild spongioform degeneration to status spongiosus (not ‘reversible per se’).
  5. PRES - T - hyperintensive encephalopathy…. reversible…
81
Q
  1. Which is not a cause of cerebellar atrophy?
  2. Phenytoin
  3. Hyperparathyroidism
  4. Carcinomatosis
  5. ETOH
  6. Seizures
A

ANSWER:2.Hyperparathyroidism - F - can cause skull vault thickening similar to phenytoin

19.Which is not a cause of cerebellar atrophy? (TW)

  1. Phenytoin - T - phenytoin toxicity predominantly affecting cerebellar hemispheres (without cerebral atrophy).
  2. Hyperparathyroidism - F - can cause skull vault thickening similar to phenytoin
  3. Carcinomatosis - T - idiopathic degeneration secondary to carcinoma (ie paraneoplastic)… usually oat cell carcinoma of lung.
  4. ETOH - T - predominantly affecting midline (vermis), without cerebral atrophy.
  5. Seizures - T - recognised complication of chronic epilepsy
82
Q
  1. Which is not associated with benign intracranial hypotension?
  2. Large pituitary gland
  3. Pachymeningeal enhancement
  4. Low lying tonsils
  5. Pial enhancement
A
  1. Pial enhancement - F - pachymeningeal enhancement.
  2. Which is not associated with benign intracranial hypotension? (TW)
  3. Large pituitary gland - T - enlarged pituitary gland.
  4. Pachymeningeal enhancement - T - diffuse dural thickening / enhancement. Pachymeninges (both supra- and infratentorial. Primarily affects inner (meningeal) layer. May extend into IACs).
  5. Low lying tonsils - T - Caudal displacement of tonsisl in 25-75%. Downward displacement of brain through incisura (“slumping” midbrain). Sagittal T1 shows brain descent in 40-50% cases.
  6. Pial enhancement - F - pachymeningeal enhancement.
83
Q
  1. Which is the least likely cause of cerebral venous thrombosis?
  2. Meningioma
  3. Post partum
  4. Pseudotumor cerebri
  5. Gastroenteritis
  6. Thyrotoxicosis
A

3.Pseudotumor cerebri

  • DV sinus thrombosis is a cause for pseudotumor cerebri
    Wide spectrum of predisposing causes (>100 identified) for cortical venous / dural venous sinus thrombosis.

Trauma, infection, inflammation, pregnancy, metabolic (dehydration, thyrotoxicosis, cirrhosis, etc), hematological (coagulopathy), collagen-vascular disorders (APLA syndrome), vasculitis (Behet).

Meningiomas may compress a dural sinus, may cause thrombosis of a sinus, may invade a dural sinus, or leave the sinus intact.(Osborn)

84
Q
  1. Which is true regarding cerebral aneurysms?
  2. PCA aneurysms can occur post trauma
  3. 25% present late with infarcts secondary to vasospasm
A
  1. 25% present late with infarcts secondary to vasospasm ?
    * AJL - I think the question is poorly written. 20-30% of people get clinically apparent vasospasm (therefore ischaemia) 4-7 days after SAH.
85
Q
  1. Which is incorrect with respect to mesial temporal sclerosis?
  2. Has disordered hippocampus
  3. Hippocampal atrophy
  4. Best seen on axial T2 weighted sequences
  5. Cortical dysplasia is most common dual pathology
A
  1. Best seen on axial T2 weighted sequences - F - Best imaging tool High resolution MR of temporal lobes, thin section coronal T2WI, angled perpendicular to long axis of hippocampus.
  2. Which is incorrect with respect to mesial temporal sclerosis? (TW, GC)
  3. Has disordered hippocampus - T - decrease in hippocampal neurons and gliosis. Chronic astrogliosis with a fine fibrillary background containing bland nuclei of astrocytes and few remaining neurons. Relative sparing of CA2 subfield.
  4. Hippocampal atrophy - T - Atrophy of mesial temporal lobe (hippocampal body 88%, hippocampal tail 61%, Hippocampal head 51%, Amygdala 12%)
  5. Best seen on axial T2 weighted sequences - F - Best imaging tool High resolution MR of temporal lobes, thin section coronal T2WI, angled perpendicular to long axis of hippocampus.
  6. Cortical dysplasia is most common dual pathology - T – developmental MTS commonly have a second lesion, usually cortical dysplasia.(Osborn)
86
Q
  1. Which is not a cause of leptomeningeal enhancement?
  2. Viral meningitis
  3. Benign intracranial hypotension
  4. Bacterial meningitis
  5. Carcinomatosis
A

2.Benign intracranial hypotension

87
Q
  1. Which is true with respect to arterial dissection?
  2. Vertebral artery dissection is more common than carotid dissection
  3. Vertebral artery dissection does not cause posterior fossa SAH
  4. Vertebral artery dissection is a recognised cause of a dural AVF
  5. A dissection flap is normally seen with ultrasound
  6. There is hyperdensity within the lumen on non contrast CT
A
  1. Vertebral artery dissection is a recognised cause of a dural AVF - ?T - Not much specifid supporting evidence for this (other than iatrogenic cause following intervention for vert dissections). Dahnert says “AVF location: - carotid-cavernous sinus fistula (most common); vertebral artery fistula. ; external carotid fistula (rare).
  2. Which is true with respect to arterial dissection? (TW)
  3. Vertebral artery dissection is more common than carotid dissection - F - spontaneous dissections involve ICA 68%, vert 27%, both 5%. Traumatic dissection ICA > VA.
  4. Vertebral artery dissection does not cause posterior fossa SAH - F - subadventital dissections can cause pseudoaneurysmal dilatation - prone to rupture through adventitia. Incidence about 5%. (eMed)
    * LW: Intracranial artery dissection may result in subarachnoid hemorrhage. In a 2015 systematic review of retrospective case series, subarachnoid hemorrhage was associated with intracranial artery dissection in 8 to 69 percent of cases.
  5. Vertebral artery dissection is a recognised cause of a dural AVF - ?T - Not much specifid supporting evidence for this (other than iatrogenic cause following intervention for vert dissections). Dahnert says “AVF location: - carotid-cavernous sinus fistula (most common); vertebral artery fistula. ; external carotid fistula (rare).
  6. A dissection flap is normally seen with ultrasound - F - an intimal flap is rarely depicted floating in the lumen or separating two lumina with different doppler signals (Radiographics 2008). US is not hte procedure for Dx of dissection because detection of the intimal flap has not been reliable (Radiographics 2005). US 50% accuracy (Dahnert).
  7. There is hyperdensity within the lumen on non contrast CT - F - usually no abnormality is seen in artery (Osborn). At UECT of the brain a spontaneous crescent hyperattenuating area corresponding hematoma may be noticed in acute dissection at the upper portions of the ICA (Radiographics 2008): - but this is intramural, not intraluminal.
88
Q
  1. A 30 y.o presents with seizures. There is T2 hyperintensity within the medial temporal lobe. What is the most likely cause?
  2. DNET
  3. Mesial temporal sclerosis
  4. Pilocytic astrocytoma
  5. Cortical dysplasia
A
  1. Mesial temporal sclerosis - T - T2 hyperintense signal in hippocampus. Disease of childhood / young adults. Most common cause of partial complex seizures.
  2. A 30 y.o presents with seizures. There is T2 hyperintensity within the medial temporal lobe. What is the most likely cause? (TW)
  3. DNET - F - Patients <20yo. Temporal lobe most common site (often amygdala / hippocampus). Very hyperintense on T2. T1 “bubbly” hypointense mass. Epilepsy most common sign/symptoms.
  4. Mesial temporal sclerosis - T - T2 hyperintense signal in hippocampus. Disease of childhood / young adults. Most common cause of partial complex seizures.
  5. Pilocytic astrocytoma - F - >80% less than 20yo. Usually cerebellum (60%) > optic nerve/chiasm (25-30%) > adjacent to 3rd ventricle > brainstem.
  6. Cortical dysplasia - F - Polymicrogyria sometimes referrd to as cortical dysplasia. PMG has predilection for perisylvian regions. Irregular, small gyri, indistinct cortical-white matter junction. hypomyelination +/- cortical infolding. T2 may not be bright. (Osborn)
89
Q
  1. Cerebral abscess. What is not an expected finding?
  2. Restricted diffusion
  3. Thick enhancing wall
  4. Absent lactate peak
  5. Absent choline peak
A
  1. Absent lactate peak - F - central necrotic area may show presence of acetate, lactate, alanine, succinate, pyruvate, and amino acids.
  2. Cerebral abscess. What is not an expected finding? (TW)
  3. Restricted diffusion - T - increased DWI in cerebritis and abscess. ADC map markedly decreased signal centrally within abscess.
  4. Thick enhancing wall - T - depends on stage: early cerebritis - patchy enhancement; late cerebritis - intense but irregular rim enhancement; early capsule - well-defined, thin walled enhancing rim; late capsule - cavit collapses, thickened enhancement of capsule.
  5. Absent lactate peak - F - central necrotic area may show presence of acetate, lactate, alanine, succinate, pyruvate, and amino acids.
  6. Absent choline peak - T - choline-containing compounds (used to make cell membranes) will be reduced. Creatine (involved in energy metabolism). N-acetyl asparate (assoc with myelin sheaths of nerve cell fibers). Spectral characteristics of intracranial abscesses - absence of NAA (2.0 ppm); abscence of choline (3.2 ppm); abscence of PCr/Cr (3.0 ppm); presence of cytosolic amino acids such as leucine, isoleucine, and valine (0.9 ppm); presence of lactate (1.3 ppm), acetate (1.92 ppm), succinate (2.4ppm), and alanine (1.5ppm); and occasionally lipids (0.8-1.2 ppm).
90
Q
  1. MR spectroscopy. Which is false?
  2. Lactate is located at 1.3 ppm
  3. Creatinine is used as a reference metabolite
  4. N-acetyl aspartate is a neuronal marker
  5. Components of cell membranes results in choline peak
A

2.Creatinine is used as a reference metabolite - F - creatine, not creatinine. Creatine is involved in energy metabolism. Creatinine is a break down product of creatine phosphate in muscle.

28.MR spectroscopy.
Which is false? (TW)

  1. Lactate is located at 1.3 ppm - T
  2. Creatinine is used as a reference metabolite - F - creatine, not creatinine. Creatine is involved in energy metabolism. Creatinine is a break down product of creatine phosphate in muscle.
  3. N-acetyl aspartate is a neuronal marker - T - NAA assoc with myelin sheaths of nerves cell fibers.
  4. Components of cell membranes results in choline peak - T - choline-containing compounds (used to make cell membranes)
91
Q
  1. Which is most correct?
  2. CT is the 1st investigation of choice for retinoblastoma
  3. 75% of children with optic gliomas have NF1
  4. A 3rd of optic nerve gliomas are calcified
  5. Trilateral retinoblastoma involves a focus in the suprasella cistern
A

1.CT is the 1st investigation of choice for retinoblastoma - T - CT provides a sensitive method for Dx and detecting intraocular calcification, and shows the tumor extent even in the absence of Ca+. US useful for Dx NB from non-neoplastic conditions. MRI not as specific as CT as not as sensitive in detecting Ca+ (eMed)

  1. Which is most correct? (TW)
  2. CT is the 1st investigation of choice for retinoblastoma - T - CT provides a sensitive method for Dx and detecting intraocular calcification, and shows the tumor extent even in the absence of Ca+. US useful for Dx NB from non-neoplastic conditions. MRI not as specific as CT as not as sensitive in detecting Ca+ (eMed)
  3. 75% of children with optic gliomas have NF1 - F - of all optic pathway gliomas 33% occur in NF1 patients.
  4. A 3rd of optic nerve gliomas are calcified - F - calcification is rarely seen (noted by 14% of cases by Hoyt et al). Calcification is more often seen in chiasmal gliomas.
  5. Trilateral retinoblastoma involves a focus in the suprasella cistern - F - Trilateral is bilateral RBs plus pineal tumor. Quadrilateral is trilateral plus 4th focus in suprasella cistern
92
Q
  1. Which is false with respect to normal myelination patterns?
  2. The optic radiations are myelinated by 4 months
  3. Myelination of the corpus callosum begins at the splenium and progresses anteriorly
  4. There is a period between 24-30 months when WM intensity is the same as GM intensity
  5. Anterior limb of the internal capsule myelinates after the posterior limb
A
  1. There is a period between 24-30 months when WM intensity is the same as GM intensity
  2. Which is false with respect to normal myelination patterns? (TW)

1.The optic radiations are myelinated by 4 months - T - Early optic radiations start at 2m, optic radiations more apparent at 4 months. Optic tracts myelinated at birth.
2.Myelination of the corpus callosum begins at the splenium and progresses anteriorly - T
3.There is a period between 24-30 months when WM intensity is the same as GM intensity F
4.Anterior limb of the internal capsule myelinates after the posterior limb - T - Posterior limb is evident at birth, anterior limb at 2 months. Myelination - Bottom to top / Back to front / Central to Peripheral. Normally beings 5th fetal month and continues throughout life.
T1 preceeds T2 in myelination. Fat in myelin - T1 hyperintensity. As myelin sheaths tighten peri-axonal water is diplaced and T2 becomes more hypointense.

93
Q
  1. Which is false with respect to chordomas?
  2. Age <30 yrs
  3. Mets in spine
  4. Most occur at ends of axial skeleton
  5. Soft tissue mass
A
  1. Age <30 yrs - F - age 30-70 (mean 50yo)
  2. Which is false with respect to chordomas? (TW)
  3. Age <30 yrs - F - age 30-70 (mean 50yo)
  4. Mets in spine - T - can have intradural spinal seeding
  5. Most occur at ends of axial skeleton - T - 50% in sacrum, 35% in clivus, 15% in vertebrae, 5% other sites.
  6. Soft tissue mass - T - soft-tissue mass - lobulated tumor contained within pseudocapsule
94
Q
  1. Which is false in neurofibromatosis?
  2. NF1 is more common than NF2
  3. Is a casue of J-shaped sella
  4. Pseudoarthrosis is a diagnostic criteria for NF2
  5. Multiple schwannomas in NF2
A
  1. Pseudoarthrosis is a diagnostic criteria for NF2 - F - See below – not criteria for NF-1 or NF-2
  2. Which is false in neurofibromatosis? (TW)
  3. NF1 is more common than NF2 - T - NF1 1:3000, NF2 1:50000
  4. Is a cause of a J-shaped sella -T - also, optic nerve glioma which can be seen in NF1
  5. Pseudoarthrosis is a diagnostic criteria for NF2 - F - See below – not criteria for NF-1 or NF-2
  6. Multiple schwannomas in NF2 - T - bilateral vestibular schwannomas is the sine qua non of NF2. Schwannoma of other cranial nerves.

Diagnostic criteria for NF2:
bilateral vestibular schwannomas;
OR 1st degree relative with NF2 and 1 vestibular schwannoma;
OR 1st degree relative with NF2 and 2 of the folloiwng: neurofibroma, meningioma, glioma, schwannoma, posterior subcapsular lenticular opacity.

NF2 - MISME : multiple inherited schwannomas, meningiomas, and ependymomas.

• Diagnostic criteria for NF-1:
two or more of the following: (NOB-FLC)
o Neurofibromas ≥ 2 or plexiform neurofibroma
o Optic pathway glioma
o Bony lesion distinctive for NF-1, e.g. sphenoid wing dysplasia, thinning of long bone cortex (with or without pseudoarthrosis)
o Freckling of axilla or inguinal region
o Lisch nodule ≥ 2 (hamartomas of the iris)
o Café-au-lait spots ≥ 6 (> 15mm in adults & > 5mm in children)
o 1st degree relative with NF-1

95
Q
  1. 18 y.o female with well defined skull lesions centered in the diploic space. Isointense T1 signal, hyperintense T2. Which is most likely?
  2. Eosinophilic granuloma
  3. Pacchionian granulations/ venous lakes
  4. Mets
  5. Fibrous dysplasia
  6. Lymphoma
A

**LJS - ?more likely they are just arachnoid granulations (*AJL agrees)

  1. Eosinophilic granuloma - T – most <5yo (but age range 2 – 30yo), “beveled edge”, “hole-within-a-hole”, “button sequestrum”
  2. 18 y.o female with well defined skull lesions centered in the diploic space. Isointense T1 signal, hyperintense T2. Which is most likely? (TW)
  3. Eosinophilic granuloma - T – most <5yo (but age range 2 – 30yo), “beveled edge”, “hole-within-a-hole”, “button sequestrum”
  4. Pacchionian granulations / venous lakes - F - common finding. Iso T1, and hyper T. PG’s communicate with dural sinus, parasagittal or within 3cm of SSS, inner table > diploe > outer table. VL’s are an outpouching of diploic vein with an irregular well-demarcated contour.
  5. Mets - F - older patients, often with history of cancer
  6. Fibrous dysplasia - F - skull involvement more with polyostotic form (which has mean age 8yo). Monostotic form can involve craniofacial bones (typically <30yo). Tend to be hypointense on T1 and T2.
  7. Lymphoma - F - less likely, + different signal characteristics?
96
Q
  1. Which is least likely involved in hypoxic encephalopathy
  2. Basal Ganglia
  3. Cortical layers 3, 4 and 6
  4. Corpus callosum
  5. Watershed
  6. Periventricular white matter in premature infants
A

3.Corpus callosum - seems least likely option

97
Q

RADIODIAGNOSIS AUGUST 2007:35.Regarding MS, which is true?

  1. 30% have plaques in cervical cord.
  2. Black hole is seen on T2 images.
  3. Plaques lie perpendicular to the axis of the corpus callosum.
  4. Lesions more juxtacortical compared to those in small vessel disease.
A
  1. Plaques lie perpendicular to the axis of the corpus callosum. T - majority are ovoid lesions oriented with long axis perpendicular to ventricular walls (Dawson’s fingers = perivenous demyelination)
  2. Regarding MS, which is true? (GC)
  3. 30% have plaques in cervical cord. F - up to 80% of all MS patients have spinal involvement. Of these, cervical cord lesions account for 2/3 of cases. ie. Around 50% have cervical cord plaques. Seen as eccentric involvement of the dorsal and lateral elements abutting the subarachnoid space.
  4. Black hole is seen on T2 images. F - well-marginated discrete foci of T2 hyperintensity (represents loss of hydrophobic myelin which results in an increase in water content)
  5. Plaques lie perpendicular to the axis of the corpus callosum. T - majority are ovoid lesions oriented with long axis perpendicular to ventricular walls (Dawson’s fingers = perivenous demyelination)
  6. Lesions more juxtacortical compared to those in small vessel disease. F - lesions may be subependymal and periventricular as in small vessel disease. MS may also involve corpus callosum, internal capsule, centrum semiovale, corona radiata, optic n./chiasm/tract, brainstem (trigeminal root entry zone), cerebellar peduncles, cerebellum. 10% of MS plaques occur in gray matter. Ddx from small vessel disease: >50 yo., lesions <5mm, not infratentorial. (Dahnert 6th, eMedicine: spine MS)
98
Q
  1. Regarding masses in HIV, which is true?
  2. PML enhances.
  3. PML spares subcortical U fibres.
  4. PML and HSV can be distinguished on DWI.
  5. Cryptococcus infection follows VR spaces.
A

4.Cryptococcus infection follows VR spaces. T = most common presentation of cryptococcous brain infection in AIDS.

  1. Regarding masses in HIV, which is true? (GC/SK)
  2. PML enhances. F - typically don’t, but faint periph enhancement has been described.
  3. PML spares subcortical U fibres. F - subcortical U fibre involvement is frequently seen.
  4. PML and HSV can be distinguished on DWI. F - PML can show restricted diffusion, although uncommon (eMedicine). Evaluation with DWI has been limited - some describe a leading edge of high SI; may correlate with speed of clinical progression (active infection as DWI abnormality; recent studies suggest that DWI may be useful for documenting response to HAART (eMedicine; RG 2008).
  5. Cryptococcus infection follows VR spaces. T = most common presentation of cryptococcous brain infection in AIDS.
99
Q
  1. Which of the following is correct in regards to tumors?
  2. SEGA does not enhance.
  3. Neurocytoma is homogenous signal on T2.
  4. Neurocytoma arises from the septum pellucidum.
  5. 90% of subependymomas arise from the 4th ventricle.
A
  1. Neurocytoma arises from the septum pellucidum. T - or the ventricular wall.
  2. Which of the following is correct in regards to tumors? (GC)
  3. SEGA does not enhance. F - markedly contrast-enhancing mass located near the foramen of Munro (distinguishes SEGA from a subependymal nodule, which does not enhance). Occurs in 6-16% of all patients with tuberous sclerosis.
  4. Neurocytoma is homogenous signal on T2. F - heterogenous due to numerous intratumoral cyst-like areas. Well-circumscribed, heavily lobulated intraventricular mass.
  5. Neurocytoma arises from the septum pellucidum. T - or the ventricular wall.
  6. 90% of subependymomas arise from the 4th ventricle. F - 2/3 infratentorial, 1/3 supratentorial (Osborn)
100
Q
  1. Features of mesial temporal sclerosis, which is false?
  2. Doesn’t enhance.
  3. Decreased NAA on MRS.
  4. Increased choline on MRS.
  5. Parahippocampal gyrus is smaller.
A
  1. Increased choline on MRS. F
  2. Features of mesial temporal sclerosis, which is false? (GC)
  3. Doesn’t enhance. T
  4. Decreased NAA on MRS. T - reflects neuronal loss in the ipsilateral temporal lobe (seizure focus). There is also a diffuse bilateral hemispheric reduction in NAA of the ipsilateral frontal, parietal and occipital lobes, thought to be due to metabolic impairment from repeated seizures rather than neuronal loss. (AJNR 2002)
  5. Increased choline on MRS. F
  6. Parahippocampal gyrus is smaller. T - decreased volume cf. contralateral side on T2WI
101
Q
  1. Regarding corpus callosal dysgenesis, which is false?
  2. Genu is always present in partial agenesis.
  3. Cingulate gyrus is normal.
  4. Medial parietal lobes are affected.
  5. Foramen of Monro is enlarged.
A
  1. Cingulate gyrus is normal. F - “sunburst gyral pattern” = dysgenesis of cingulate gyrus with characteristic radial orientation of cerebral sulci from the roof of the 3rd ventricle. Also see persistent eversion of cingulate gyrus (rotated inferiorly and laterally) with absence on midsagittal images.
  2. Regarding corpus callosal dysgenesis, which is false? (GC)
  3. Genu is always present in partial agenesis. T - partial agenesis is milder form and depends on time of arrested growth. Anteroposterior development of genu then body then splenium, with rostrum forming last) - thus variations all include the genu.
  4. Cingulate gyrus is normal. F - “sunburst gyral pattern” = dysgenesis of cingulate gyrus with characteristic radial orientation of cerebral sulci from the roof of the 3rd ventricle. Also see persistent eversion of cingulate gyrus (rotated inferiorly and laterally) with absence on midsagittal images.
  5. Medial parietal lobes are affected. T - “Probst bundles” are longitudinally-oriented WM tracts that run along the medial wall of the lateral ventricles and end randomly in occipital and parietal lobes.
  6. Foramen of Monro is enlarged. T - with a high-riding 3rd ventricle. Characteristic imaging features: axial: parallel lateral ventricles with colpocephaly (racing car) coronal: ‘trident-shaped’ appearance of frontal horns with high-riding 3rd ventricle and ‘keyhole’ temporal horns, Probst bundles
102
Q
  1. Regarding haemangioblastomas, which is false?
  2. Most common intraparenchymal primary infratentorial malignancy in adults.
  3. Commonly calcifies.
  4. Usually paravermian in location.
  5. Associated with erythrocytosis.
A
  1. Commonly calcifies.
  2. Regarding haemangioblastomas, which is false? (GC)
  3. Most common intraparenchymal primary infratentorial malignancy in adults. T - accounts for 10% of posterior fossa tumours.
  4. Commonly calcifies. F - calcification is rare. Classic appearance is a cystic lesion with an enhancing mural nodule, with nodule abutting the pial surface. 10% are solid enhancing lesions, 15% are enhancing lesions with multiple cystic areas.
  5. Usually paravermian in location. T - paravermian cerebellar location in 85% > spinal cord > cerebral hemisphere/brainstem. Multiple lesions in 10% - think vHL.
  6. Associated with erythrocytosis. T - in 20%, tumour elaborates EPO (Dahnert, Primer)
103
Q
  1. In regards to HSV infection, which is false?
  2. Spares the basal ganglia.
  3. Is low density on CT.
  4. Shows no mass effect.
  5. Involves the insular cortex.
A

3.Shows no mass effect. F - mild mass effect with compression of the lateral ventricles and loss of Sylvian fissure (brain oedema).

  1. In regards to HSV infection, which is false? (GC)
  2. Spares the basal ganglia. T - spared putamen forms a sharply defined concave/straight border. Locations: inferomedial temporal > frontal > parietal lobes; propensity for limbic system; initially predominantly unilateral.
  3. Is low density on CT. T - may be negative in first 3 days; poorly defined areas of mildly decreased attenuation.
  4. Shows no mass effect. F - mild mass effect with compression of the lateral ventricles and loss of Sylvian fissure (brain oedema).
  5. Involves the insular cortex. T
104
Q

RADIODIAGNOSIS APRIL 2007:42.Craniopharyngioma, which is true?

  1. Peripheral nodular calcification in up to 90% of paediatric cases
  2. Predominantly solid in children
  3. Usually cystic in adults
  4. Predominantly infrasellar
  5. Rarely causes bony destruction of sella
A
  1. Peripheral nodular calcification in up to 90% of paediatric cases T - 90% rule for childhood craniopharyngiomas (90% cystic, 90% calcified). Adamantinomatous subtype is more common in kids - path recapitulates the structure of the enamel organ of a tooth: wet keratin, Ca2+, cysts filled with cholesterol (sump oil).
  2. Craniopharyngioma, which is true? (GC)
  3. Peripheral nodular calcification in up to 90% of paediatric cases T - 90% rule for childhood craniopharyngiomas (90% cystic, 90% calcified). Adamantinomatous subtype is more common in kids - path recapitulates the structure of the enamel organ of a tooth: wet keratin, Ca2+, cysts filled with cholesterol (sump oil).
  4. Predominantly solid in children F - see above
  5. Usually cystic in adults F - Papillary subtype more common in adults, this lacks keratin, calcification and cysts. Only 30-50% of adult tumours have calcification.
  6. Predominantly infrasellar F - 70% are intra/suprasellar, 20% suprasellar, 10% intrasellar, may also arise in stalk / tuber cinereum. Ectopic craniopharyngioma is rare - floor of 3rd ventricle or sphenoid bone.
  7. Rarely causes bony destruction of sella F - may be associated with extensive sellar destruction, evident in 75% of skull xrays. Added “predominantly’” to b., changed c. from “calcification in 90% in adults”, added option e.
105
Q
  1. Agenesis of the Corpus callosum, which is least likely associated?
  2. Holoprosencephaly
  3. Midline arachnoid cyst
  4. Chiari I malformation
  5. Hydrocephalus
  6. Trisomy
A
  1. Chiari I malformation
    * LW: associated with Chiari II.

Other associated anomalies: DW malformation, midline lipoma, midline encephalocoele, polymicrogyria, GM heterotopia, hypotelorism median cleft syndrome, CVS / GIT / GU anomalies (62%).

Aetiology:
Sporadic
Syndromic - trisomy, DW spectrum, Chiari
Congenital insult: fetal alcohol exposure, CMV infection (Dahnert, Pocket Rad paeds neuro & obs)

106
Q
  1. Toxoplasmosis of brain, which is false?
  2. Always causes microcephaly
  3. Haemorrhage may occur following therapy
  4. Most common CNS infection in AIDS patients
  5. Periventricular calcification in neonates
  6. Most common in basal ganglia
A
  • *LJS edit: 3. is also false. HIV encephalophalitis is the most common CNS infection (direct infection of CNS lymphocytes/microglial cells) by HIV
  • AJL: Robbins - “Toxoplasma encephalitis, due to reactivation, is the most common opportunistic pathogen of the CNS” so maybe depends on the wording and the other options available (not disagreeing just providing some additional wording that might help)
  1. Always causes microcephaly F
  2. Toxoplasmosis of brain, which is false? (GC)
  3. Always causes microcephaly F - Microcephaly & mental retardation are sequelae of Toxo-ORCH infection (Osborn). May see macrocephaly due to hydrocephalus (from aqueductal stenosis); head size may return to normal or remain enlarged. Dahnert contradicts himself on this (pg 310 & 330).
  4. Haemorrhage may occur following therapy T - haemorrhage and/or calcifications may occur
  5. Most common CNS infection in AIDS patients T - occurs in up to 40% of AIDS patients.
  6. Periventricular calcification in neonates T - diffuse calcification; also seen in thalamus, BG, choroid plexus. If only PV calcification, then more likely to be CMV or tuberous sclerosis.
  7. Most common in basal ganglia T - 75%; also scattered throughout brain parenchyma at GW jxn.
107
Q
  1. Cerebral vascular lesions, which is true?
  2. 90% are detected on CT
  3. Angiography detects cavernomata
  4. Vein of Galen aneurysm is an AVM
  5. Venous angiomas bleed more commonly in posterior fossa than anteriorly
  6. AVMs are most commonly dural-based
A
  • ** LJS edit*** vein of Galen malformation are an AV fistula, no nidus. ?Bad recall - previous versions of this question were: “90% cerebral vascular lesions are detected on CT” which is true
  • LW: agrees option 3 is false and is actually a fistula.
  1. Vein of Galen aneurysm is an AVM T - consists of a central AVM directly draining into a secondarily enlarged VOG (hence ‘aneurysm’ is a misnomer), drains brain AND the malformation. cf. VOG malformation = dilated prosencephalic vein of Markowski (drains only the malformation), persistent AV flow retards embryonic vein involution & prevents development of the true VOG; most common extracardiac cause of high-output CHF in neonate. SK – F – Osborn calls it an AV fistula. SG agrees
  2. Cerebral vascular lesions, which is true? (GC/SK)
  3. 90% of cavernous malformations are detected on CT F - NECT is negative in 30-50%, may be hyperattenuating or calcified, minimal or intense enhancement, minimal mass effect or surrounding oedema. MRI: popcorn or mulberry appearance, usually with a bright lobulated centre (T1 and T2) and a T2 hypointense (haemosiderin) rim. Susceptibility sequence necessary to detect coexistent smaller lesions (multiple cavernomas in 60-80%).SK – T – as per above responses to this question.
  4. Angiography detects cavernomas F - usually normal due to slow flow (“crytpic” or “ occult”).
  5. Vein of Galen aneurysm is an AVM T - consists of a central AVM directly draining into a secondarily enlarged VOG (hence ‘aneurysm’ is a misnomer), drains brain AND the malformation. cf. VOG malformation = dilated prosencephalic vein of Markowski (drains only the malformation), persistent AV flow retards embryonic vein involution & prevents development of the true VOG; most common extracardiac cause of high-output CHF in neonate. SK – F – Osborn calls it an AV fistula. SG agrees
  6. Venous angiomas bleed more commonly in posterior fossa than anteriorly F - haemorrhage is rare, however, there is an association with cavernomas (which can bleed). One theory regarding this association is the elevated venous pressures (medusa head draining into one large vein) which may help produce the secondary cavernomas.
  7. AVMs are most commonly dural-based F - 80% parenchymal (usually congenital; ICA & vertebral a. supply), 10% dural (mostly acquired; ECA supply), 10% mixed. (Osborn, Primer, Dahnert)
108
Q
  1. DWI, which is true?
  2. Vasogenic oedema shows restriction
  3. Infarct bright on ADC
  4. Altered diffusion signal around abscess
  5. Extracellular methaemoglobin is bright on DWI
A
  1. Extracellular methaemoglobin is bright on DWI T - ADC is bright after lysis of RBC membranes, therefore extracellular metHb appears bright on T2 and ADC, and brightness on DWI is due to T2 shine-through.
  2. DWI, which is true? (GC + TW)
  3. Vasogenic oedema shows restriction F - Vasogenic oedema, caused by incompetence of blood vessels or breakdown of the BBB, is an increase in the amount of extracellular water, thus no restriction. Cytotoxic oedema implies damage to cell membranes, reflecting failure of Na-K pump with cell swelling, therefore restricted motion of H+ within cells.
  4. Infarct bright on ADC F - dark on ADC, bright on DWI.
  5. Altered diffusion signal around abscess F - necrotic area harbours a complex matrix of proteins, inflam cells, cellular debris, and bacteria in high viscosity pus. Water molecules are bound to carboxyl, hydroxyl and AA groups on the surface of macromolecules. These characteristics contribute to restricted Brownian motion within the cavity.
  6. Extracellular methaemoglobin is bright on DWI T - ADC is bright after lysis of RBC membranes, therefore extracellular metHb appears bright on T2 and ADC, and brightness on DWI is due to T2 shine-through.

ICH evolution from oxyHb (hyperacute, 4-6hrs)

  • deoxyHb (acute, 7-72hrs)
  • metHb (subacute, 1-4wks)
  • haemosiderin (chronic, >4wks).

Oxyhaemoglobin is diamagnetic with no unpaired electrons (ie. no T1 or T2 shortening) - appears bright on T2 and DWI, but ADC is low (due to intact RBC membranes).

DeoxyHb is paramagnetic with 4 unpaired electrons (significant T2 shortening), and is shielded from H2O by globin (prevents T1 shortening).

Intracellular metHb, haemosiderin and ferritin are also paramagnetic and generally hypointense on T2 and DWI. AJR 2003.

109
Q
  1. Ethesioneuroblastoma, which is false
  2. Believed to arise from olfactory epithelium
  3. Appears hypointese to isointense on T2WI
  4. Unilateral nasal obstruction is the most common symptom
  5. Treatment is based generally on surgery and radiotherapy only
A

LJS edit 2. probably correct - highly cellular tumours with heterogeneous intermediate signal on T1 and T2

  • LW: agree with LJS re RP stating heterogenous T1 and T2 signal mass, so hopefully un recalled option was more false.
    4. more likely to be false - rx is surgery plus chemo (is small round blue cell tumour!) +/- radiotherapy
    2. Appears hypointese to isointense on T2WI - F - hypointense on T1, and isointense or hyperintense on T2WI

47.Ethesioneuroblastoma, which is false? (TW)
1.Believed to arise from olfactory epithelium - T
2.Appears hypointese to isointense on T2WI - F - hypointense on T1, and isointense or hyperintense on T2WI
3.Unilateral nasal obstruction is the most common symptom - T - unilateral nasal obstruction 70%, epistaxis 46%.
4.Treatment is based generally on surgery and radiotherapy only - T - low-grade and low-stage tumors should be treated by surgery, followed, in most cases, by radiation therapy.
*LW:
The role of chemotherapy, either before or after RT or surgery, is not established. Numerous studies have evaluated various chemotherapy regimens in an effort to improve outcomes [80-88]. However, it is unclear whether this actually improves results compared with a combined craniofacial resection and RT, it is however commonly included in order to try and improve outcomes.

110
Q

RADIODIAGNOSIS AUG 200648.Objective evidence of pulsatile tinnitus

  1. foramen spinosum enlarged with persistent stapedial artery
  2. MRI moderately sensitive for Menieres D
  3. L transverse sinus is larger than the right
  4. Dehiscent jugular bulb into middle ear
  5. High riding jugular bulb into the middle ear
A
  1. Dehiscent jugular bulb into the middle ear T – one of the causes of pulsatile tinnitus – typically “humming”
  2. Objective evidence of pulsatile tinnitus (JS)
  3. foramen spinosum enlarged with persistent stapedial artery (absent) F – absent f spinosum is associated with persistent stapedial artery
  4. MRI moderately sensitive for Menieres disease F – Menieres is a audiological/clinical diagnosis
  5. L transverse sinus is larger than the right (R>L)
  6. Dehiscent jugular bulb into the middle ear T – one of the causes of pulsatile tinnitus – typically “humming”
  7. High riding jugular bulb F – not associated with PT unless dehiscent

Radiological features to look for with pulsaltile tinnitus – dehiscent jugular bulb or carotid, absent f spinosum, AVM, benign intracranial hypertension, dural AVF, vascular compression of CN VIII, glomus tympanicum or jugulare

111
Q
  1. 60y.o Male, Dumbell shaped lesion centered on the cribriform plate with extension into the anterior cranial fossa superiorly and into the nasopharynx inferiorly, the lesion most likely represents:
  2. Esthesioneuroblastoma
  3. Lymphoma
  4. Nasopharyngeal carcinoma
  5. SCC
  6. Adenoid cystic carcioma
A

1.Esthesioneuroblastoma T – Neuroendocrine malignancy of neural crest origin that arises from the olfactory epithelium; dumbbell shaped mass in the anterior intracranial fossa and nasal cavity with waist at the level of the cribriform plate

112
Q
  1. 40 y.o Chinese male, soft tissue density filling nasopharynx with bony destruction:
  2. Esthesioneuroblastoma
  3. Lymphoma
  4. Nasopharyngeal carcinoma
  5. SCC
  6. Adenoid cystic carcinoma
A

3.Nasopharyngeal carcinoma T - SCC arising from the nasopharyngeal mucosal space, increased risk in Chinese and Eskimos, relationship to EBV, diet and genetic factors

113
Q
  1. With regard to acquired cholesteatoma:
  2. The commonest site of tympanic perforation is the pars flaccida
  3. Prussak’s space is rarely involved
  4. The scutum is typically eroded at its tip
  5. Extension into the mastoid antrum is via the tectum
  6. Gadolinium DTPA enhancement on MRI is a typical finding
A
  1. The commonest site of tympanic perforation is the pars flaccida T – most common when referring to acquired cholestetoma. Pars tensa is most common when referring to just the tympanic membrane.
    or3. The scutum is typically eroded at its tip T
  2. With regard to acquired cholesteatoma: (JS, GC, TW)
  3. The commonest site of tympanic perforation is the pars flaccida T – most common when referring to acquired cholestetoma. Pars tensa is most common when referring to just the tympanic membrane.
  4. Prussak’s space is rarely involved F – Pars flacida cholesteatoma occurs in Prussak’s space, lateral to head of malleus
  5. The scutum is typically eroded at its tip T
  6. Extension into the mastoid antrum is via the tectum F – Extension into the antrum is via the aditus ad antrum. Dehiscence of the tegmen tympani
  7. Gadolinium DTPA enhancement on MRI is a typical finding F – no enhancement of CT itself, may see rim enhancement on MR which is related to granulation tissue (Harnsberger)
114
Q
  1. Regarding vestibular schwannomas:
  2. Bilateral tumours are associated with NF1
  3. Haemorrhage is uncommon
  4. About 70% are cystic
  5. Meningeal reaction is typical
  6. A melanotic variant is a recognized entity
A
  1. Haemorrhage is uncommon T – 0.5% have haemorrhagic foci on MR (Harnsberger)
  2. Regarding vestibular schwannomas: (JS)
  3. Bilateral tumours are associated with NF1 F – associated with NF2
  4. Haemorrhage is uncommon T – 0.5% have haemorrhagic foci on MR (Harnsberger)
  5. About 70% are cystic F – 15% show intramural cysts (Harnsberger)
  6. Meningeal reaction is typical F – as opposed to meningioma
  7. A melanotic variant is a recognized entity F – very very rare variant
115
Q
  1. 03.52 The following may cause both intraconal and extraconal lesions in the orbit:
  2. Metastatic breast carcinoma
  3. Orbital Pseudotumour
  4. Orbital varix
  5. Mikulicz’s syndrome
  6. Dermoid tumour
A
  1. Metastatic breast carcinoma T – lymphoma and mets can occur in any compartmentor
  2. Orbital Pseudotumour T – idiopathic inflammatory disease of the orbit; can occur in any area of the orbit and can cross anatomic boundaries (Harnsberger)
  3. 03.52 The following may cause both intraconal and extraconal lesions in the orbit: (JS)
  4. Metastatic breast carcinoma T – lymphoma and mets can occur in any compartment
  5. Orbital Pseudotumour T – idiopathic inflammatory disease of the orbit; can occur in any area of the orbit and can cross anatomic boundaries (Harnsberger)
  6. Orbital varix F – tends to be intraconal (Harnsberger) with uniform enhancement and enlargement on Valsalva
  7. Mikulicz’s syndrome F – non-specific swelling of the salivary and lacrimal glands. Can be observed in association with TB, sarcoid, syphilis and lymphoma

5.Dermoid tumour F - typically occurs in the anterior extraconal orbit;UOQ 60%, UIQ 25%.
Hansberger says they can be pre or post septal.

Intraconal lesions: glioma, meningioma, haemangioma, pseudotumur, lymphoma, mets
Extraconal lesions: cellulits/abscess, lymphoma, mets, dermoid, lymphangioma, lacrimal gland tumours
Conal lesions: pseudotumour, thyroid, rhabdo

116
Q
  1. Focal destruction within the petrous bone is a typical manifestation of, T/F:
  2. Pituitary tumour
  3. Cholesterol granuloma
  4. Epidermoid of the cerebello-pontine angle
  5. Inflammatory cholesteatoma
  6. Glomus jugulare tumour
A

Cholesterol granuloma, inflammatory choleastoma, glomus jugulare

54.Focal destruction within the petrous bone is a typical manifestation of, T/F: (JS)

  1. Pituitary tumour F – not within petrous apex; can cause erosion of the sphenoid
  2. Cholesterol granuloma T – expansile mass within the petrous apex with trabecular breakdown and cortical thinning and dehiscence
  3. Epidermoid of the cerebello-pontine angle F – within the CPA and doesn’t erode into the petrous temporal bone
  4. Inflammatory cholesteatoma T – Mass in the middle ear with bony erosion
  5. Glomus jugulare tumour T – paraganglioma within the jugular foramen (formed by occipital and petrous temporal bones) with permeative bony destruction on CT
117
Q
  1. The following are true in relation to cerebellopontine angle tumours:
  2. Type 2 neurofibromatosis is associated with multiple schwannomas
  3. Acoustic neuromas are typically hyperintense on TI weighted images
  4. Peripheral enhancement is a feature of epidermoid cysts
  5. Choroid plexus seen in the cerebellopontine angle is a normal feature
  6. Acoustic neuromas most commonly arise from the cochlear portion of the acoustic nerve
A
  1. Type 2 neurofibromatosis is associated with multiple schwannomas T – bilateral acoustics, meningiomas, gliomas, schwannomas
  2. The following are true in relation to cerebellopontine angle tumours: (JS)
  3. Type 2 neurofibromatosis is associated with multiple schwannomas T – bilateral acoustics, meningiomas, gliomas, schwannomas
  4. Acoustic neuromas are typically hyperintense on TI weighted images F – typically low on T1 with strong contrast enhancement
  5. Peripheral enhancement is a feature of epidermoid cysts F – do not enhance, high on T2, low on T1
  6. Choroid plexus seen in the cerebellopontine angle is a normal feature F – can occur but ??? normal
  7. Acoustic neuromas most commonly arise from the cochlear portion of the acoustic nerve F – arise from vestibular nerve in >90%
118
Q
  1. With regard to the parapharyngeal space, the following are true:
  2. It abuts the infratemporal fossa
  3. The contents are displaced posteriorly by a mass arising in the pharyngeal mucosa
  4. The contents are displaced medially by a parotid space mass
  5. It extends from the skull base to the hyoid
  6. It contains mostly lymphoid tissue
A
  1. It abuts the infratemporal fossa T – continuous with the intratemporal fossa anterosuperiorlyor
  2. The contents are displaced medially by a parotid space mass T or
  3. It extends from the skull base to the hyoid T – inverted pyramid from the skull base to the greater horn of the hyoid
  4. With regard to the parapharyngeal space, the following are true: (JS)
  5. It abuts the infratemporal fossa T – continuous with the intratemporal fossa anterosuperiorly
  6. The contents are displaced posteriorly by a mass arising in the pharyngeal mucosa F – displaced laterally by a lesion from the mucosal surface
  7. The contents are displaced medially by a parotid space mass T
  8. It extends from the skull base to the hyoid T – inverted pyramid from the skull base to the greater horn of the hyoid
  9. It contains mostly lymphoid tissue F – contains fat, V3 and internal maxillary artery
119
Q
  1. The following statements regarding agenesis of the corpus callosum are true:
  2. Ventriculomegaly commonly affects the temporal horns
  3. Is associated with Down’s syndrome
  4. The lateral ventricles are widely placed
  5. There is an association with heterotopic gray matter
  6. It is associated with colpocephaly
A
  1. The lateral ventricles are widely placed T
  2. There is an association with heterotopic gray matter T
  3. It is associated with colpocephaly T
  4. The following statements regarding agenesis of the corpus callosum are true: (JS)
  5. Ventriculomegaly commonly affects the temporal horns F – typically develop dilatation of the occipital horns
  6. Is associated with Down’s syndrome F – not a common association that I can find (*LW trisomy 18, 13, and 8)
  7. The lateral ventricles are widely placed T
  8. There is an association with heterotopic gray matter T
  9. It is associated with colpocephaly T
120
Q

SEPTEMBER 200558.Intraspinal tumours, which is true:

  1. Gliomas enhance uniformly
  2. Meningiomas most commonly arise in the cervical spine
  3. Meningiomas are hyperintense to cord on T2
  4. Nerve sheath tumours are hyperdense to disc on CT
  5. Haemanagioblastomas are associated with prominent veins
A
  1. Haemanagioblastomas are associated with prominent veins - T - nonglial highly vascular discrete nodular masses abutting leptomeninges with prominent dilated and tortuous vessels on posterior cord surface. Assoc with vHL. Majority intramedullary (75%). Thoracic cord 50% > cervical cord 40%.
  2. Intraspinal tumours, which is true: (TW)
  3. Gliomas enhance uniformly - F - patchy irregular Gd-enhancement on MR (astrocytoma). Ependydmoma enhance homogeneously in 84%. Patchy (65%, / no (15%0 tumor enhancement in gangliogliomas of spinal cord.
  4. Meningiomas most commonly arise in the cervical spine - F - thoracic 82%, cervical on anterior cord surface near foramen magnum is 2nd most common. 50% are intradural extramedullary.
  5. Meningiomas are hyperintense to cord on T2 - F - isointense with spinal cord T1 and T2.
  6. Nerve sheath tumours are hyperdense to disc on CT - F - low attenuation due to high lipid content of myelin from Schwann cells / entrapped fat / endoneural myxoid tissue with high water content (Antoni B areas).
  7. Haemanagioblastomas are associated with prominent veins - T - nonglial highly vascular discrete nodular masses abutting leptomeninges with prominent dilated and tortuous vessels on posterior cord surface. Assoc with vHL. Majority intramedullary (75%). Thoracic cord 50% > cervical cord 40%.
121
Q

59.Lumbar spine, which is most correct:

  1. Spondylolisthesis at L4 most commonly associated with spondylolysis
  2. Disc hernation is most commonly posterolateral
  3. Degenerative changes most prominent mid lumbar spine
  4. Height of disc is directly proportional to the size of the protrustion
  5. Limbus vertebra is of superior end-plate
A
  1. Disc hernation is most commonly posterolateral - T - herniation of nucleus pulposus: posterolateral is weakest point (49%) as PLL tightly adherent to posterior margins fo disk. Posterocentral 8%, lateral / foraminal (<10%).
  2. Lumbar spine, which is most correct: (TW)
  3. Spondylolisthesis at L4 most commonly associated with spondylolysis - F - L5 on S1 most commonly spondylolysis (ie open arch). pars defects L5 70-90%, L4 15-30%, L3 1-2%. L4 on L5 most commonly degenerative (closed arch / pseudospondylolisthesis)
  4. Disc hernation is most commonly posterolateral - T - herniation of nucleus pulposus: posterolateral is weakest point (49%) as PLL tightly adherent to posterior margins fo disk. Posterocentral 8%, lateral / foraminal (<10%).
  5. Degenerative changes most prominent mid lumbar spine - F - degenerative disc disease is seen most commonly in lumbar spine at L4-L5 and L5-S1, and in the cervical spine at C5-C6 and C6-C7.
  6. Height of disc is directly proportional to the size of the protrustion - F - narrowing of intervertebral disk space may be seen, but is most commonly a sign of disk degeneration.
  7. Limbus vertebra is of superior end-plate - F - LV is from an intrabody herniation of disc material at junction of vertebral bony rim of central and endplate (anterosuperior corner). Can occur anterioinferior margin of cervical spine. Anterior herniation of nucleus pulposus may cause a separation of a triangular smooth bone fragment which apparently represents the ring apophysis.
122
Q

60.Regarding haemangioblastomas, which is false

  1. Most common intraparenchymal primary infratentorial malignancy in adults
  2. Commonly calcifies
  3. Usually paravermian location
  4. Associated with erythrocytosis
  5. Associated with phaeochromocytoma
A
  1. Commonly calcifies - F - almost never calcifies
  2. Regarding haemangioblastomas, which is false (TW)
  3. Most common intraparenchymal primary infratentorial malignancy in adults - T - most common primary infratentoria neoplasm in adults. >80% in adulthood avg 33y. <20% in childhood: in vHL.
  4. Commonly calcifies - F - almost never calcifies
  5. Usually paravermian location - T - paravermian cerebellar hemisphere 85% > spinal cord > cerebral hemisphere
  6. Associated with erythrocytosis - T - erythrocythemia in 20% as tumor elaborates stimulant (EPO)
  7. Associated with phaeochromocytoma - T - Assoc with vHL (in 20%). Phaeo (often familial). Syringomyelia. Spinal cord hemangioblastomas.
123
Q

62.Multiple sclerosis, which is true?

  1. up to 30% in spinal cord only
  2. “black hole” is best seen on T2WI
  3. enhancing lesions characteristically last less than 3 months
  4. lesions in the spinal cord are usually perpendicular to the long axis of the cord
A
  1. up to 30% in spinal cord only - T - 12-33% involvement of spinal cord without coexistent intracranial plaques (Dahnert).
  2. Multiple sclerosis, which is true? (TW)
  3. up to 30% in spinal cord only - T - 12-33% involvement of spinal cord without coexistent intracranial plaques (Dahnert).
  4. “black hole” is best seen on T2WI - F – T1 black holes
  5. enhancing lesions characteristically last less than 3 months - F - >90% of enhancement disappears in 6/12
  6. lesions in the spinal cord are usually perpendicular to the long axis of the cord - F - atrophic plaques are orinetated along spinal cord asix. Length of plaque usually less than 2 vertebral body segments. Plaques are orientated perpendicular to the corpus callosum in the brain (perpendicular calloseptal T2 hyperintensities). (Dahnert; MRI atlas of spine)
124
Q
  1. Neurofibromatosis type I, which is false?
  2. Plexiform neurofibromatosis
  3. Multiple meningiomas
  4. Transmitted on chromosome 17
  5. Dural ectasia
A
  1. Multiple meningiomas - F - NF2 - MISME (multiple inherited schwannomas, meningiomas, and ependymomas).
  2. Neurofibromatosis type I, which is false? (TW)
  3. Plexiform neurofibromatosis - T
  4. Multiple meningiomas - F - NF2 - MISME (multiple inherited schwannomas, meningiomas, and ependymomas).
  5. Transmitted on chromosome 17 - T - von Recklinghausen = 17 letters. NF2 - Ch 22.
  6. Dural ectasia - T

NF1 = peripheral NF (90%). 1:3000 (cf NF2 1: 50000). Phakomatoses.
Diagnostic clinical criteria
- at least 2 must be present:
>/= 6 cafe-au-lait spots (>5mm prepurbetal, >15mm postpubertal individuals)
- >/= 2 neurofibromas of any type / >/= 1 plexiform neurofibroma axillary / inguinal freckling optic pathway glioma
>/= 2 Lisch nodules (= pigmented iris hamartomas)
- characteristic skeletal lesion: sphenoid bone dysplasia, dysplasia + thinning of long bone cortex
- 1st degree relative (parent, sibling, child) with NF1. CME 2001

125
Q
  1. Central neurocytoma ?which is false
  2. Elderly
  3. Calcification
  4. Enhances
  5. Histo similar to oligo
A
  1. Elderly - F - typically 20-40 yrs (account for 75%). Represents 50% of intraventricular tumours in this age group
  2. Central neurocytoma ?which is false (TW)
  3. Elderly - F - typically 20-40 yrs (account for 75%). Represents 50% of intraventricular tumours in this age group
  4. Calcification - T - Calcification common 50-70%.
  5. Enhances - T - CECT moderate, heterogeneous enhancement. MRI Gd moderate to strong heterogeneous enhancement.
  6. Histo similar to oligo - T – fried-egg cells and previously was thought to be oligidendroglioma. Electron microscopy and immunohisto revealed neuronal lineage of the tumor cells

Central neurocytoma
- Intraventricular neuroepithelial tumor with neuronal differentiation. “Bubbly” mass in frontal horn or body of lateral ventricle.
Typically supratentorial, intraventricular (attached to septum pellucidum).
>50% in frontal horn/body of lateral ventricle near foramen of Monro.
Ca+ common (50-70%).
Mixed solid and cystic.
Can be rarely associated with medulloblastoma. Usually benign, local recurrence uncommon. Sx resection typically curative. (Osborn)

126
Q
  1. Acute Herpes simplex encephalitis, which is false?
  2. Spares the basal ganglia
  3. Is low density on CT
  4. Shows no mass effect
  5. Involves the insular cortex
A
  1. Shows no mass effect - F - Mass effect. Swollen cortex / subcortical white matter.
  2. Acute Herpes simplex encephalitis, which is false? (TW)
  3. Spares the basal ganglia - T - basal ganglia usually spared
  4. Is low density on CT - T - often normal early, then low attentuation, mild mass effect in medial temporal lobes, insula.
  5. Shows no mass effect - F - Mass effect. Swollen cortex / subcortical white matter.
  6. Involves the insular cortex - T - typically medial temporal and inferior lobes.

Limbic system: temporal lobes, insula, subfrontal area and cingulate gyri typical. Involvement of cingulate gyrus, contralateral temporal lobe highly suggestive. HSV-1 causes 95% of all herpetic encephalitis.
HSV-2 more common in neonates.
Haemorrhagic, necrotizing encephalitis of gray and white matter (primarly limbic system). May se mild patchy enhancement. DWI - may see restricted diffusion. CME

127
Q

66.Regarding adrenoleukodystrophy, which is least correct

  1. There is enhancement
  2. Asymmetrical involvement
  3. Periatrial and splenium of the corpus callosum involved
  4. Affects the white matter, adrenals, and testicles
A
  1. Asymmetrical involvement - F – usually symmetrical
  2. Regarding adrenoleukodystrophy, which is least correct (TW)

1.There is enhancement - T – leading edge (intermediate zone) enhancement. Contrast enhancement strongly linked to progression. CECT can show linear enhancement of intermediate zone.
2.Asymmetrical involvement - F – usually symmetrical
3.Periatrial and splenium of the corpus callosum involved - T - Classically involves peritrigonal white matter. Pattern: Splenium - peritrigonal WM - corticospinal tracts / fornix / commisural fibers / visual and auditory pathways.
4.Affects the white matter, adrenals, and testicles - T –
VLCFA accumulate in the
i) CNS myelin
ii) Adrenal cortex
iii) Leydig cells of the testes
3-10yo.
X-Linked recessive.
Defective peroxisomal fatty acid oxidation due to impaired function of lignoceryl-CoA ligase
- get accumulation of saturated very long chain fatty acids in white matter and adrenal cortex and testes.

128
Q

67.In relation to diffuse axonal injury of the brain, which is true?

  1. The ventral brain stem is commonly affected
  2. Most lesions are haemorrhagic
  3. The genu is the portion of the corpus callosum most often involved
  4. 80% of diffuse axonal lesions are not visible on CT
  5. Lesions characteristically occur within the cortex
A
  1. 80% of diffuse axonal lesions are not visible on CT - T - 80% of all DAI lesions are microscopic, nonherhagic. Visible lesions are “tip of the iceberg”.
  2. In relation to diffuse axonal injury of the brain, which is true? (TW)
  3. The ventral brain stem is commonly affected - F - brainstem involved, esp dorsolateral midbrain and upper pons.
  4. Most lesions are haemorrhagic F – 20% haemorrhagic - F - Hyperdense foci of petechial haemorrhage seen in 20-50% of CTs.
  5. The genu is the portion of the corpus callosum most often involved - F - CC involved 20% cases - 3/4 involve splenium / undersurface of posterior body.
  6. 80% of diffuse axonal lesions are not visible on CT - T - 80% of all DAI lesions are microscopic, nonherhagic. Visible lesions are “tip of the iceberg”.
  7. Lesions characteristically occur within the cortex - F - common locations are gray/white matter interface 67%, corpus callosum 20% (majority splenium / undersurface of posterior body), brainstem (esp dorsolateral midbrain and upper pons).
129
Q
  1. Medulloblastoma, which is true?
  2. resistant to radiotherapy
  3. on CT calcification helps to differentiate from pilocytic astrocytoma .
  4. desmoplastic variant occurs in children younger than 3 years old
  5. spinal metastases occur in the posterior spinal canal more than anterior
  6. most bone metastases are lytic lesions
A
  1. spinal metastasis occur in the posterior spinal canal more than anterior - T- as normal flow of CSF from the cisterna magna travels first along the posterior margin of the spinal cord before returning to the cistern along the ventral surface of the spinal cord, most mets are found along the posterior maring of the spinal cord.
  2. Medulloblastoma, which is true? (TW)
  3. resistant to radiotherapy - F - Rx with surgery and radiotherapy (extremely radiosensitive)
  4. on CT calcification helps to differentiate from pilocytic astrocytoma - F - Ca+ in up to 20% in both MB and pilocytic astrocytomas.
    * LW: I would say this partly true, but less correct than above, as per Donnelly: “unlike medulloblastoma, pilocytic astrocytoma usually does not demonstrate Ca++.
  5. desmoplastic variant occurs in children younger than 3 years old - F - MB in adult patients are often the desmoplastic histologic type, & prone to recurrence (Radiographics03)
  6. spinal metastasis occur in the posterior spinal canal more than anterior - T- as normal flow of CSF from the cisterna magna travels first along the posterior margin of the spinal cord before returning to the cistern along the ventral surface of the spinal cord, most mets are found along the posterior maring of the spinal cord.
  7. most bone metastases are lytic lesions - F - osseous lesions are usually sclerotic (65% cases, lytic in 35% of cases). Bone is the most common extraneural metastatic site (adults and children). Followed by lymphnodes (33%). Also asked in Apr 05. Radiographics article.
130
Q

APRIL 200470.01.05 Which of the following is TRUE with regard to head ultrasound in the infant:

  1. The normal choroid plexus is a homogeneous hypoechoic structure
  2. Parenchymal haemorrhage and infarction can be readily differentiated
  3. The normal cerebellum is less echogenic than the supratentorial cortex
  4. It is the test of choice for possible extracerebral haemorrhage prior to fontanella closure
  5. Parenchymal calcification in congenital infection may be seen on ultrasound earlier than on CT scan
A
  1. Parenchymal calcification in congenital infection may be seen on ultrasound earlier than on CT scan - T - CME says true. Some studies say US = CT sensitivity indetecting the Ca+. I guess when thinking about kidney and nephrocalcinosis - US more sensitive and CT more specific indetection of nephrocalcinosis…. extrapolate?
  2. 01.05 Which of the following is TRUE with regard to head ultrasound in the infant: (TW)
  3. The normal choroid plexus is a homogeneous hypoechoic structure - F - hyperechoic
  4. Parenchymal haemorrhage and infarction can be readily differentiated - F - both can appear quite echogenic
  5. The normal cerebellum is less echogenic than the supratentorial cortex - F - cerebellum appears as a hyperechoic structure behind the pons in the posterior cerebral fossa on sagittal plane US.
  6. It is the test of choice for possible extracerebral haemorrhage prior to fontanella closure - F - limited examination of extracerebral areas due to limited windows / skull.
  7. Parenchymal calcification in congenital infection may be seen on ultrasound earlier than on CT scan - T - CME says true. Some studies say US = CT sensitivity indetecting the Ca+. I guess when thinking about kidney and nephrocalcinosis - US more sensitive and CT more specific indetection of nephrocalcinosis…. extrapolate?
131
Q

71.With regard to medulloblastomas, which is incorrect:

  1. 75% occur laterally in cerebellum
  2. The earlier the age of presentation in the paediatric age group the worse the prognosis
  3. Calcification is less common than ependymomas
  4. The adult variety of medulloblastoma demonstrates cystic change in about 80%
  5. Subarachnoid seeding is a feature
A
  1. 75% occur laterally in cerebellum - F - vermis cerebellli + roof of 4th ventricle (younger age group) in 91%, cerebellar hemisphere in older age group.
  2. With regard to medulloblastomas, which is incorrect: (TW)
  3. 75% occur laterally in cerebellum - F - vermis cerebellli + roof of 4th ventricle (younger age group) in 91%, cerebellar hemisphere in older age group.
  4. The earlier the age of presentation in the paediatric age group the worse the prognosis - T - adult presentation slightly better outcome (may reflect greater resectability of lateral lesions, desmoplastic variant)
  5. Calcification is less common than ependymomas - T - calcification in 20% of MB. Calcification in ependymoma 50%. *LW agrees.
  6. The adult variety of medulloblastoma demonstrates cystic change in about 80% - T -
  7. Subarachnoid seeding is a feature - T - subarachnoid metastatic spread (30-100%) via CSF pathway to spinal cord and cauda equina.
132
Q
  1. Regarding vestibular schwannomas
  2. bilateral tumors are associated with NF1
  3. Haemorrhage is uncommon
  4. About 70% are cystic
  5. Meningeal reaction is typical
  6. A melanotic variant is a recognised entity
A
  1. Haemorrhage is uncommon - T - hemorrhagic vestibular schwannomas are rare entities. When happens has been shown to present with acute neurological changes and deterioration.
  2. Regarding vestibular schwannomas: (TW)
  3. bilateral tumors are associated with NF1 - F - NF2 (MISME - multiple inherited schwannomas, meningiomas, ependymomas)
  4. Haemorrhage is uncommon - T - hemorrhagic vestibular schwannomas are rare entities. When happens has been shown to present with acute neurological changes and deterioration.
  5. About 70% are cystic - F - 15% have intramural cysts (Harnsberger)
  6. Meningeal reaction is typical - F

5.A melanotic variant is a recognised variant - F - CME says F. However there are a handful of cases reported (so very, very rare).
Melanotic schwannoma is a rare melanin-producing nerve sheat tumor.

MSs are of neural crest origin, prob from neoplastic proliferation of a common precursor cell for both Schwann cells and melanocytes.
Most involve cranial or spinal nerve roots (esp spinal Cx and thoracic nerves). 10% involve symp chain. There have been reports of MSs of vestibular nerves. MSs may be component of Carney complex.

VS is most common neoplasm of internal auditory canal / CP angle.
Cellular dense region = Antoni A. Loose areas with widely separated cells = Antoni B.
Arises from within IAC in 80% / cochlea. CP angle at opening of IAC (porus acusticus) with intracanalicular extension in 5%. 85% from vestibular portion of 8th CN. 15% from cochlear portion. CME 03.19

133
Q
  1. Multiple sclerosis, which is true?
  2. upto 30% in spinal cord only
  3. “black hole” is best seen on T2WI
  4. enhancing lesions characteristically last less than 3 months
  5. liesions in the spinal cord are usually perpendicular to the long axis of the cord
  6. lesions more juxtacortical compared to those in small vessel disease
A
  1. up to 30% in spinal cord only - T - 12-33% involvement of spinal cord without coexistent intracranial plaques (Dahnert).
  2. Multiple sclerosis, which is true? (TW + poached from GC)
  3. up to 30% in spinal cord only - T - 12-33% involvement of spinal cord without coexistent intracranial plaques (Dahnert).
  4. “black hole” is best seen on T2WI - F – T1 black holes
  5. enhancing lesions characteristically last less than 3 months - F - >90% of enhancement disappears in 6/12
  6. lesions in the spinal cord are usually perpendicular to the long axis of the cord - F - atrophic plaques are orinetated along spinal cord asix. Length of plaque usually less than 2 vertebral body segments. Plaques are orientated perpendicular to the corpus callosum in the brain (perpendicular calloseptal T2 hyperintensities). (Dahnert; MRI atlas of spine)
  7. Lesions more juxtacortical compared to those in small vessel disease. F - lesions may be subependymal and periventricular as in small vessel disease. MS may also involve corpus callosum, internal capsule, centrum semiovale, corona radiata, optic n./chiasm/tract, brainstem (trigeminal root entry zone), cerebellar peduncles, cerebellum. 10% of MS plaques occur in gray matter.
134
Q

74.Ependymomas in the paediatric age group

  1. Occur only in the infratentorial intracranial compartment
  2. Occur in both benign and malignant histological forms
  3. Calcify in less than about 10% of cases
  4. In the lumbar region arise in the nerve roots
A
  1. Occur in both benign and malignant histological forms - T – Microscopic features: ependymoma - perivascular pseudorosettes. Anaplastic ependymoma - high cellularity, nuclear atypia, hyperchromatism. Occasional pseudopalisadin or necrosis in most malignant lesions. Staging: WHO II low grade, well differentiated; WHO III high grade, anaplastic.
  2. Ependymomas in the paediatric age group (TW)
  3. Occur only in the infratentorial intracranial compartment - F – Infratentorial (floor of 4th ventricle) 70% of all intracranial ependymomas. Can occur supratentorial: frontal > parietal > temporoparietal > justaventricular. Conus (40-65% of all spinal intramedullary gliomas. In children (intracranial compartment) 70% are infratentorial, 30% supratentorial.
  4. Occur in both benign and malignant histological forms - T – Microscopic features: ependymoma - perivascular pseudorosettes. Anaplastic ependymoma - high cellularity, nuclear atypia, hyperchromatism. Occasional pseudopalisadin or necrosis in most malignant lesions. Staging: WHO II low grade, well differentiated; WHO III high grade, anaplastic.
  5. Calcify in less than about 10% of cases - F – Calcification common 50% in both supra- and infra-tentorial types.
  6. In the lumbar region arise in the nerve roots - F - originates from ependymal cells lining the central canal. Ependymoma of spinal cord is the most common intramedullary spinal neoplasm in adults. Cervical cord only 44% > thoracic cord alone 26% > conus medullaris 7%.
135
Q
  1. With regard to epidermoid tumors of the CNS, which is incorrect:-
  2. they generally do not enhance following intravenous contrast media injection
  3. they have an irregular, multilobulated contour
  4. in most cases they have a similar signal to CSF
  5. they are best detected in heavily T2WI
A
  1. they are best detected in heavily T2WI - F - often isointense (65%) to slightly hyperintense (35%) to CSF on T2. Usually doesn’t completely null on FLAIR. DWI definitively distinguished from arachnoid cyst.
  2. With regard to epidermoid tumors of the CNS, which is incorrect:- (TW)
  3. they generally do not enhance following intravenous contrast media injection - T - usually no enhancement (CECT) and T1C+. However margin of cyst may show minimal enhancement (35%).
  4. they have an irregular, multilobulated contour - T - lobulated, irregular, “cauliflower-like” mass with ‘fronds’. Location - 90% are intradural, primarilyin basal cisterns: CPA 40-50%, 4th ventricle 17%, parasellar 10-15%. Rarely in cerebral hemispheres 1.5%.
  5. in most cases they have a similar signal to CSF - T - “dirty CSF” appearance. Uncommonly hypointense to CSF (“black epidermoid”).
  6. they are best detected in heavily T2WI - F - often isointense (65%) to slightly hyperintense (35%) to CSF on T2. Usually doesn’t completely null on FLAIR. DWI definitively distinguished from arachnoid cyst.
136
Q

76.Colloid cysts, which is true?

  1. Majority are hypodense on unenhanced CT
  2. Majority are hyperintense T1
  3. Can sometimes see central enhancementon T1 C+ sequences.
  4. Vast majority are symptomatic
  5. 50% eventually stop enlarging or remain stable
A
  1. Majority are hyperintense T1 - T - 2/3 are hyperintense on T1, 1/3 isointense. Signal correlates with cholesterol concentration.
  2. Colloid cysts, which is true? (TW)
  3. Majority are hypodense on unenhanced CT - F - 2/3 are hyperdense. 1/3 iso/hypodense. Density correlations inversely with hydration state.
  4. Majority are hyperintense T1 - T - 2/3 are hyperintense on T1, 1/3 isointense. Signal correlates with cholesterol concentration.
  5. Can sometimes see central enhancement on T1 C+ sequences - F - rarely may show peripheral (rim) enhancement
  6. Vast majority are asymptomatic - F - 40-50% are asymptomatic, discovered incidentally. 8% of these patients will develop cyst-related symptoms by 10y.
  7. 50% eventually stop enlarging or remain stable - F - 90% stable or stop enlarging (usually in the older age group, small cysts with no hydrocephalus, hyperdense on NECT, hypointense on T2). 10% enlarge (usually younger patients, larger cysts, iso/hypodense on NECT, often hyperintense on T2).
137
Q
  1. The following statements regarding suprasellar masses in adults are true:
  2. Craniopharyngiomas exhibit calcification in children about 90%
  3. Arachnoid Cyst pushed the Stalk forward
  4. Craniopharyngiomas show less increase in pituitary fossa size than adenomas
  5. Normal height of the pituitary gland in post menopausal women 15mm
A
  1. Craniopharyngiomas exhibit calcification in children about 90% - T - Adamantinomatous type (children) 90% calcify, 90% mixed solid (iso-), cystic (hypodense).. Papillary (squamous papillary) type (adult) rarely calcifies, often solid, isodense. Both types - 90% enhance.
  2. The following statements regarding suprasellar masses in adults are true: (TW)
  3. Craniopharyngiomas exhibit calcification in children about 90% - T - Adamantinomatous type (children) 90% calcify, 90% mixed solid (iso-), cystic (hypodense).. Papillary (squamous papillary) type (adult) rarely calcifies, often solid, isodense. Both types - 90% enhance.
  4. Arachnoid Cyst pushed the Stalk forward - F - 10% arachnoid cysts are suprasellar & would push stalk back. 10% ACs are in cerebellopontine angle, 50-60% in middle cranial fossa.
  5. Craniopharyngiomas show less increase in pituitary fossa size than adenomas - F - variable size, often large at presentation (>5cm).
  6. Normal height of the pituitary gland in post menopausal women 15mm - F - normal pit measurements: children 6mm, males and post-menopausal females 8mm, young menstruating females 10mm, pregnant, lactating females 12mm.
138
Q
  1. With regards to Childhood Craniophayngiomas, which is false?
  2. Adamantinomatous type more common
  3. Lack of calcification and enhancement more supportive of rathke cleft cyst
  4. WHO grade II lesion
  5. Most common suprasellar mass in children
A
  1. WHO grade II lesion - F - WHO grade I
  2. With regards to Childhood Craniophayngiomas, which is false? (TW)
  3. Adamantinomatous type more common - T - more common in children. Squamous-papillary type more common in adult. Ad ~50%, Papillary ~30%, mixed ~20%.
  4. Lack of calcification and enhancement more supportive of rathke cleft cyst - T - ~90% of craniopharyngiomas in children have calcification, and 90% show enhancement. RCCs are noncalcified, and usually don’t enhance. Small RCC maybe indistinguishable from intrasellar CP.
  5. WHO grade II lesion - F - WHO grade I
139
Q

79.Cerebral cavernous angiomata:, whic is true?

  1. Usually increased signal on DWI
  2. Are angiographically occult
  3. Have an internal elastic lamina in their walls
  4. Are uniformly bright on T2 weighted MR images
A
  1. Are angiographically occult - T - conventional DSA is usually normal “angiographically occult vascular malformation”, slow intralesional flow without AV shunting. Rare to get venous pooling, contrast “blush”.
  2. Cerebral cavernous angiomata, which is true? (TW)
  3. Usually increased signal on DWI - F - usually normal
  4. Are angiographically occult - T - conventional DSA is usually normal “angiographically occult vascular malformation”, slow intralesional flow without AV shunting. Rare to get venous pooling, contrast “blush”.
  5. Have an internal elastic lamina in their walls - F - discrete collection of endothelial-lined, hemorrhage-filled vessels without intervening normal brain. Thin-walled epithelial-lined spaces, embedded in collagenous matrix.
  6. Are uniformly bright on T2 weighted MR images - F - Reticulated “popcorn-like” lesion is most typical. Mixed signal core, complete hypointense hemosiderin rim. Locules of blood with fluid-fluid levels. Hypointese lesion possible, but less commonAngiogenically immature lesions with endothelial proliferation, increased neoangiogenesis. 75% sporadic. 10-30% multiple, familial (Multiple cavernous malformation - autosomal dominant)Associations: DVA, superficial siderosis, cutaneous abnormalities (cafe au lait spots, Hyperkeratotic capillary-venous malformations “cherry angiomas”)UECT negative in 30-50%. 40-60% calcify.Peak presentation 40-60yo, but may presetn in childhood. Added option 1.
140
Q

EPTEMBER 2002 TRUE/FALSE80.Chiari II malformation associations, which of the following is correct (T/F)?

  1. A concave petrous temporal bone
  2. Midline defect in occipital squama
  3. Deformity of the tectum of the mid brain
  4. Fenestration of the falx
  5. “Straberry” shaped head is seen on antenatal US
A

1,3,4 is true

  1. Chiari II malformation associations, which of the following are correct (T/F)? (TW)
  2. A concave petrous temporal bone - T - “scalloped” petrous pryamid, “notched” clivus
  3. Midline bond defect in occipital squama - F - Chiari III.
  4. Deformity of the tectum of the mid brain - T – small posterior fossa with “towering” cerebellum proturding through incisura - compresses midbrain, causes “beaked” tectum.
  5. Fenestration of the falx - T - dural abnormality. Fenestrated/hypoplastic falx with interdigitated gyri.
  6. “Strawberry” shaped head is seen on antenatal US - F - see banana sign or lemon head. Strawberry shaped head is reflective of T18 (brachycephaly, frontal narrowing / hypoplasia of frontal bones), can also see in thanatophoric dwarfism.
141
Q

81.Diffuse Axonal Injury, which option is most correct?

  1. Is most common in the ventral brainstem
  2. Is most common in the genu of the corpus callosum
  3. Is often hemorrhagic
  4. 80% of cases are not shown on CT
  5. Occurs in the cortex.
A
  1. 80% of cases are not shown on CT - T - 80% microscopic, nonhemorrhagic lesions.
  2. Diffuse Axonal Injury, which option is most correct? (TW)
  3. Is most common in the ventral brainstem - F - dorsolateral midbrain and upper pons
  4. Is most common in the genu of the corpus callosum F – splenium and underside of body. 67% DAI occurs in GM/WM interface with 20% involving corpus callosum.
  5. Is often haemorrhagic F/T – 20-50% see haemorrhagic foci on CT scan – less correct, but would still be true if a T/F question.
  6. 80% of cases are not shown on CT - T - 80% microscopic, nonhemorrhagic lesions.
  7. Occurs in the cortex - F - occurs at gray matter / white matter inteface.
142
Q

82.Extradural haematomas (T/F)

  1. Cross sutures
  2. Are bilateral in 5%
  3. Are more likely to be venous in children than in adults
  4. Arachnoid tears cause hypodense subdural collection
  5. Diffuse axonal injury is most common in the corpus callosum
A

true, 2, 3, 4

  1. Extradural haematomas (T/F) (TW)
  2. Cross sutures - F - Does not cross sutures unless sutural diastasis/fracture present
  3. Are bilateral in 5% (2-10% adults, rare in children) - T - >95% are unilateral.
  4. Are more likely to be venous in children than in adults - T - venous epidural hematoms are most offten seen in the pediatric population are are not as often associated with skull fractures.
    * LW:
    - -> In older children and adolescents, hemorrhage most commonly arises from lacerated branches of meningeal arteries that are injured by fractured bone edges.
    - -> In infants and young children, hemorrhage more often follows disruption of emissary venous channels in the bone, large venous sinuses in the dura, or from multiple bleeding sources on the dural surface.
    - -> Infants and young children have a higher frequency of venous EDH due to the abundance of dural and diploic vasculature in areas of rapid bone growth.
  5. Arachnoid tears cause hypodense subdural collection T
  6. Diffuse axonal injury is most common in the corpus callosum - F - gray/white matter interface (67%). CC 20% of which 3/4 involve the splenium/undersurface of the posterior body. Brainstem.
143
Q

83.Cerebral Lymphoma (T/F)

  1. Occurs as a solitary mass in immunocompetent patients
  2. Is low density on CT
  3. Is low signal on T2 weighted images
  4. Is multiple in 90% in HIV
  5. Shows ring enhancement in HIV
A

*LW:
true 1 and 3 and 5

  1. Cerebral Lymphoma (T/F) (TW)
  2. Occurs as a solitary mass in immunocompetent patients - T - (CME 02.72 Danhert 57% commonly a large discrete solitary lesions - then proceeds ot say small + symmetric multiple nodular lesions 43-81%).
  3. Is low density on CT - F – classically hyperdense. May be isodense. +/- haemorrhage, necrosis in immunocompromised / HIV patients.
  4. Is low signal on T2 weighted images - T - homogeneous isointense/hypointesne to cortex in immunocompetent patients - due to high nuclear to cytoplasmic ratio. May be heterogeneous iso/hypointense in immunocompromised from hemorrhage, necrosis.
  5. Is multiple in 90% in HIV = possibly true: (Cant find numbers): Radiopedia states: in setting of immunodefiecny CNC lymphoma, the tumours are more likely to be heterogenous peripheral enhancement and multifocal.* LW
  6. Shows ring enhancement in HIV = true: immunocompromised show peripheral enhancement with central necrosis or homogeneous enhancement. Immunocompetent show strong homogenous enhancement. *LW
144
Q

84.Acute Herpes Simplex Encephalitis (T/F)

  1. Spares the basal ganglia
  2. Is low density on CT
  3. Shows no mass effect
  4. Involves the insular cortex
  5. Shows no contrast enhancement
A

true 1, 2, 4

  1. Acute Herpes Simplex Encephalitis (T/F) (TW)
  2. Spares the basal ganglia - T - Basal ganglia usually spared. Typical location - limbic system: temporal lobes, insula, subfrontal area and cingulate gyri. Cerebral convexity and posterior occipital cortex may become involved. Note that atypical patterns are seen in infants and children.
  3. Is low density on CT - T - low attenuation, mild mass effect in medial temporal lobes. Haemorrhage is typically a late feature.
  4. Shows no mass effect - F - mild mass effect
  5. Involves the insular cortex - T - limbic system: temporal lobes, insula, subfrontal area, cingulate gyri
  6. Shows no contrast enhancement - F - CT - patchy or gyriform enhancement of tempor lobes, a late feature. MRI may see mild, patchy enhancement early. Gyriform enhancement usually seen 1 week after initial symptoms.
145
Q
  1. Regarding Diffusion Weighted Imaging:
  2. Vasogenic oedema shows decreased diffusion
  3. The signal around cerebral abscesses is of restricted diffusion
  4. CVAs are high signal on ADC maps
  5. Arachnoid cysts have impaired diffusion
  6. Intracellular methaemoglobin is dark on DWI
A
  1. Intracellular methaemoglobin is dark on DWI T - deoxyHb, intracellular metHb, haemosiderin and ferritin are all paramagnetic and generally hypointense on T2 and DWI. Intracellular metHb will be low on ADC as RBC membranes are intact.
  2. Regarding Diffusion Weighted Imaging: (GC) see Q5, April 2007
  3. Vasogenic oedema shows decreased diffusion F - high on DWI, high on ADC (ie. T2 shine-thru).
  4. The signal around cerebral abscesses is of restricted diffusion F - restricted within abscess cavity.
  5. CVAs are high signal on ADC maps F - high on DWI, low on ADC (ie. true restriction).
  6. Arachnoid cysts have impaired diffusion F - follows CSF on all sequences (suppresses on FLAIR and no restricted diffusion on DWI or ADC).
  7. Intracellular methaemoglobin is dark on DWI T - deoxyHb, intracellular metHb, haemosiderin and ferritin are all paramagnetic and generally hypointense on T2 and DWI. Intracellular metHb will be low on ADC as RBC membranes are intact. (TW) ANJR 2005 -

restriction of diffusion is present in early intracranial hematomas without parenchymal infarction. Importantly, the reduction in ADC values did not significantly differ b/w the different stages of the hematomas (hyperacute - acute - early subacute - late subacute). Expected corresponding DWI hyperintensity was not always observed - reflecting factors other than ADC values influence the signal intensity on DWI. ie DWI is not a simple map of diffusion motions, but is a composite of contributions from diffusion, T1 and T2 effects.

146
Q
  1. Epidermoids of the CNS:
  2. FLAIR is the best sequence to differentiate from an arachnoid cyst
  3. Are best seen with heavily T2 weighted images
  4. Are of similar intensity to CSF
  5. Are commonly small at presentation
  6. Usually occur in midline and do not enhance
A
  1. Are of similar intensity to CSF T - majority are “black epidermoids”; which are isointense to CSF, but they are heterogenously hypointense on T1 and hyperintense on T2 (due to cholesterol in solid crystalline state + keratin within tumour +CSF within tumour interstices). The “white epidermoid” is rare; hyperintense on T1 and isointense on T2 (due to presence of triglycerides + polyunsaturated fatty acids). May also be hypointense on T2 (very rare), due to calcification, low hydration, viscous secretion, paramagnetic iron-containing pigment.
  2. Epidermoids of the CNS: (GC)
  3. FLAIR is the best sequence to differentiate from an arachnoid cyst F - DWI is most useful - epidermoid is bright (restricted diffusion due to presence of keratin and cholesterol produced by desquamation of the squamous epithelium lining the mass). FLAIR may differentiate when the SI is very similar to that of CSF; generally shows higher SI in epidermoids, with suppression (low SI) in arachnoid cysts.
  4. Are best seen with heavily T2 weighted images F - constructive interference in the steady-state or 3D fast spin-echo sequencees, which produce heavily T2-weighted images and thus excellent contrast btn CSF and all other structures, are necessary to precisely assess the extent of the tumour.
  5. Are of similar intensity to CSF T - majority are “black epidermoids”; which are isointense to CSF, but they are heterogenously hypointense on T1 and hyperintense on T2 (due to cholesterol in solid crystalline state + keratin within tumour +CSF within tumour interstices). The “white epidermoid” is rare; hyperintense on T1 and isointense on T2 (due to presence of triglycerides + polyunsaturated fatty acids). May also be hypointense on T2 (very rare), due to calcification, low hydration, viscous secretion, paramagnetic iron-containing pigment.
    * *LJS - depends on what sequence! Not similar to CSF on FLAIR (incomplete suppression - this and DWI are ways to ddx from arachnoid cyst).
  6. Are commonly small at presentation F - slow-growing tumours (due to continued desquamation), become symptomatic in adulthood when large. Tumours encase and surround nerves & arteries in the cisterns; symptoms include facial pain, CN palsies from CPA epidermoids, hydrocephalus in suprasellar lesions, chemical meningitis in middle cranial fossa lesions (due to leakage into SAS).
  7. Usually occur in the midline and do not enhance F - epidermoids are most often located off the midline at the skull base, may show rim enhancement (due to the presence of granulation tissue). Dermoids are characteristically midline (most often inferior vermis or at the vallecula), SI follows that of fat, and do not enhance unless infected.
147
Q

87.With regard to Cerebral Aneurysms:

  1. In children are associated with trauma and infection
  2. Multiple aneurysms are strongly associated with male gender
  3. A truly azygos anterior cerebral artery is associated with lobar holoprosencephaly and saccular aneurysms
  4. Are multiple in 15% to 20%
  5. Occur in the posterior circulation in 40%
A
  1. In children are associated with trauma and infection T - Intracranial aneurysms are uncommon in children and account for less than 2% of all cases. Aneurysms in the pediatric age group are often more posttraumatic or mycotic than degenerative and have a slight male predilection. Aneurysms found in children are also larger than those found in adults, avg 17mm in diameter (eMedicine). Multiple true answers actually?1, 3, 4
  2. With regard to Cerebral Aneurysms: (GC)
  3. In children are associated with trauma and infection T - Intracranial aneurysms are uncommon in children and account for less than 2% of all cases. Aneurysms in the pediatric age group are often more posttraumatic or mycotic than degenerative and have a slight male predilection. Aneurysms found in children are also larger than those found in adults, avg 17mm in diameter (eMedicine).
  4. Multiple aneurysms are strongly associated with male gender F - Intracranial aneurysms are multiple in 10-30% of all cases. About 75% of patients with multiple intracranial aneurysms have 2 aneurysms, 15% have 3, and 10% have more than 3. A strong female predilection is observed with multiple aneurysms. Although the overall female-to-male ratio is 5:1, the ratio rises to 11:1 in patients with more than 3 aneurysms (eMedicine).
  5. A truly azygos anterior cerebral artery is associated with lobar holoprosencephaly and saccular aneurysms T - alobar holoprosencephaly may be divided into two types: the azygos artery type which has an unpaired anterior cerebral artery with bilateral middle cerebral arteries, and the undifferentiated artery type with a single undulated main trunk supplying the whole cerebrum (Neuroradiology 1974). Not sure about the aneurysm bit….
  6. Are multiple in 15% to 20% T
  7. Occur in the posterior circulation in 40% F – Anterior circulation 85%. Posterior fossa 15% (basilar bifurcation 7%, basilar trunk 3%, vertebral-PICA 3%, PCA 2%) [Dahnert]
148
Q

88.Cerebral angiography: which one is false

  1. The supraclinoid internal carotid artery is best seen in a caudal (Waters) projection
  2. The origin of the opthalmic artery is intradural in the majority of people
  3. Internal carotid injections are of 8mls at 2mls per second
  4. Cavernous haemangiomata are angiographically occult
  5. The enhancement of meningiomas persist late into the venous phase
A
  1. The supraclinoid internal carotid artery is best seen in a caudal (Waters) projection F
  2. Cerebral angiography: (GC)
  3. The supraclinoid internal carotid artery is best seen in a caudal (Waters) projection F
  4. The origin of the opthalmic artery is intradural in the majority of people T
  5. Internal carotid injections are of 8mls at 2mls per second T
  6. Cavernous haemangiomata are angiographically occult T
  7. The enhancement of meningiomas persist late into the venous phase T
149
Q

APRIL 200290.Which of the following is the least likely cause of cerebral venous thrombosis?

  1. Meningioma
  2. Post partum
  3. Pseudotumor cerebri
  4. Gastroenteritis
  5. Thyrotoxicosis
A
  1. Pseudotumor cerebri - DV sinus thrombosis is a cause for pseudotumor cerebri
  2. Which of the following is the least likely cause of cerebral venous thrombosis? (TW)
  3. Meningioma
  4. Post partum
  5. Pseudotumor cerebri - DV sinus thrombosis is a cause for pseudotumor cerebri
  6. Gastroenteritis
  7. Thyrotoxicosis

Added option 3. to this qu. other options are all true.
Meningioma can cause it by mass effect or invasion.

Gastroenteritits maybe is false, but if it causes dehydration is then true. Depends how many degrees of separation are considered reasonable. Wide spectrum of predisposing causes (>100 identified) for cortical venous / dural venous sinus thrombosis. Trauma, infection, inflammation, pregnancy, metabolic (dehydration, thyrotoxicosis, cirrhosis, etc), hematological (coagulopathy), collagen-vascular disorders (APLA syndrome), vasculitis (Behet). Meningiomas may compress a dural sinus, may cause thrombosis of a sinus, may invade a dural sinus, or leave the sinus intact.(Osborn)

150
Q
  1. Regarding Herpes encephalitis, which option is true:
  2. In distribution of MCA
  3. Effects frontal and temporal lobe cortex
  4. Haemorrhage seen in more than 50% early in course of disease
  5. Calcification is a feature of the healing phase
  6. The basal ganglia are involved in more than 10%
A
  1. Effects frontal and temporal lobe cortex - T
  2. Regarding Herpes encephalitis, which option is true: (TW)
  3. In distribution of MCA - F - limbic system: temporal lobes, insula, subfrontal area and cingulate gyri typical.
  4. Effects frontal and temporal lobe cortex - T
  5. Haemorrhage seen in more than 50% early in course of disease - F - haemorrhage is typically a late feature.
  6. Calcification is a feature of the healing phase - F
  7. The basal ganglia are involved in more than 10% - F - basal ganglia usually sparedCME 02.55.
151
Q

93.Which of the following can cause ring enhancing CNS lesion (T/F):

  1. Vesicular neurocysticercosis
  2. PML
  3. Toxoplasmosis
  4. Herpes encephalitis
  5. Coccidiomycosis
A

1,3, and 5

  1. Which of the following can cause ring enhancing CNS lesion (T/F):
  2. Vesicular neurocysticercosis T - although no enhancement is typical, may see mild enhancement of the cyst wall. Ring enhancement more typical of colloidal stage.
  3. PML F – no mass effect, no haemorrhage, no enhancement
  4. Toxoplasmosis - T
  5. Herpes encephalitis - F - gyriform enhancement afte 1wk. May see mild patchy enhancement initially, not ring enhancement.
  6. Coccidiomycosis - T
152
Q
  1. Enhancing suprasellar mass usually (T/F):
  2. Craniopharyngioma
  3. Giant aneurysm
  4. Rathke’s cleft cyst
  5. Epidermoid
A

1,2 T94.Enhancing suprasellar mass usually (T/F): (TW)

  1. Craniopharyngioma - T - Adamantinomatous type (most common; adults) 90% enhance.
  2. Giant aneurysm - T - unless thrombosed
  3. Rathke’s cleft cyst - F - CECT doesn’t enhance. T1 +C no internal enhancement (+/- enhancing rim of compressed pituitary)
  4. Epidermoid - F - no enhancement is the rule. Typically occurs in posterior fossa - CPA 40%, 4th ventricle. Suprasellar 14%.
153
Q
  1. Regarding MRI spectroscopy (False):
  2. Lactate increased in XRT
  3. choline increased in tumours
  4. decreased NAA in infarcts
  5. decreased NAA in Canavan’s disease
A

1 T
2 T
3 T
4 F

  1. Regarding MRI spectroscopy (T/F): (TW, GC)
  2. Lactate increased in XRT - T - can be elevated. MRS shows elevated lactate in patients who have received radiation doses of 40Gy or more to the brain.
  3. choline increased in tumours - T – increased choline is the marker to differentiate recurrent tumor from radiation changes
  4. decreased NAA in infarcts - T - elevated lacate, decrease NAA.

4.decreased NAA in Canavan’s disease - F - elevated NAAMRS helps in differentiating radiation necrosis from recurrent/residual tumor by demonstrating severely depressed levels of NAA, choline, and creatine in radiation necrosis.
*LW: markedly elevated NAA and NAA:creatine ratio are pathognomonic for the condition,
this can be remembered using the mnemonic CaNAAvan

Tumor recurrence shows increased choline/NAA, elevated choline/creatine, and the presence of lactate.

Choline-containing compounds: used to make cell membranes. Creatine: involved in energy metabolism. N-acetyl asparate: assoc with myelin sheaths of nerve cell fibers.

154
Q

SEPTEMBER 2003(from Brendan Steinforth)96.CMV encephalitis (T/F)

  1. Characteristic inclusions
  2. Ependymal and subependymal spread
  3. May cause haemorrhage
  4. Most common intrauterine infection
  5. 50% of CMV infected fetuses are symptomatic at birth
A
  1. T
  2. T
  3. F
  4. T
  5. F
  6. CMV encephalitis (T/F) (TW)
  7. Characteristic inclusions - T - prominent cytomegatic cells with intranuclear and intracytoplasmic inclusions can be readily identified.
  8. Ependymal and subependymal spread - T - may affect any cell type by striking tendency for virus to localise in the ependymal and subependymal regions of brain
  9. May cause haemorrhage - F - necrotizing inflammatory process
  10. Most common intrauterine infection - T
  11. 50% of CMV infected fetuses are symptomatic at birth - F - 90% asymptomatic or subclinical. 5-10% symptomatic.

Double stranded DNA virus with replication inside cell nucleus causing a lytic productive / latent infection. Mamber of Herpes viridae family. Double stranded DNA virus with replication inside cell nucleus causing a lytic productive / latent infection. Mamber of Herpes viridae family.

155
Q
  1. CJD and variant CJD
  2. caused by a slow virus
  3. CJD patients live for a few months whilst vCJD patients live for a few years
  4. Associated with frontal atrophy
  5. “Pulvinar” sign is characteristic to CJD, not vCJD
A
  1. CJD patients live for a few months whilst v CJD patients live for a few years - T - mean duration illness for vCJD 14months, for CJD 4-5months. Both diseases are progressive and uniformly fatal.
  2. CJD and variant CJD (T/F) (TW)
  3. caused by a slow virus - F - rapidly progressing, fatal, potentially transmissible dementing disorder caused by a prion (proteinaceous infectious particle devoid of DNA and RNA)
  4. CJD patients live for a few months whilst v CJD patients live for a few years - T - mean duration illness for vCJD 14months, for CJD 4-5months. Both diseases are progressive and uniformly fatal.
  5. Associated with marked frontal atrophy - F - predominantly gray matter: BG, thalami, cerebral cortex (most commonly frontal and temporal lobes). White matter usually not involved. Mild atrophy. NECT may show rapidly progressive atrophya dn ventricular dilatation. Cerebral atrophy on MRI.
  6. “Pulvinar” sign is characteristic to CJD, not vCJD - F - seen in both.

vCJD can be distinguished from cases of typical sporadic CJD by:
younger age at onset of symptoms (~30yo),
less rapid progression of illness,
differences in clinical presentation and course,
differences in neuropathology.

CJD best diagnostic clue: progressive T2 hyperintensity of basal ganglia, thalamus, and cerebral cortex. “Pulvinar” sign: bilateral symmetrical hyperintensity of pulvinar (posterior) nuclei of thalamus.

156
Q
  1. Atypical scenario (which is false)
  2. craniophayrngioma in a 42 year old
  3. anaplastic thyroid cancer in a 29 year old
  4. bowel cancer in a 32 year old
A
  1. anaplastic thyroid cancer in a 29 year old – F
  2. atypical scenario
  3. Craniopharyngioma in a 42 year old – T – bimodal distribution (papillary in adult usu >50yo)
  4. anaplastic thyroid cancer in a 29 year old – F
  5. bowel cancer in a 32 year old – T – although CRC peak incidence in 60-70yo, in younger patient would look for IBD / polyposis syndromes.
157
Q

99.Pilocytic astrocytomas (T/F)

  1. Associated with NF2
  2. 50% are solid
  3. Prognosis is less than 70 % 5 year survival
  4. Hemorrhage is common
  5. Tumor may spread through subarachnoid space
A
  1. F
  2. T
  3. F
  4. F
  5. T
  6. Pilocytic astrocytomas (T/F) (TW)
  7. Associated with NF2 - F - NF1. 15% of NF1 patients develop PAs, most commonly in optic pathway. Up to 1/3 of patients with optic pathway PAs have NF1.
  8. 50% are solid - T - 50% nonenhancing cyst with strongly enhancing mural nodule. 40% solid with necrotic center, heterogeneous enhancement. 10% solid, homogeneous enhancement (Osborn). Dahnert say 33% solid.
  9. Prognosis is less than 70 % 5 year survival - F - median survival at 20y is >70%

.4.Hemorrhage is common - F - rare

5.Tumor may spread through subarachnoid space - T - can occur in rare cases, but still WHO I.

158
Q
  1. DNET (T/F)
  2. intracortical
  3. degeneration of an anaplastic astrocytoma
  4. cystic change and haemorrhage
  5. minimal or no mass effect
  6. majority contain calcification
A
  1. T
  2. F
  3. F
  4. T
  5. F
  6. DNET (T/F) (TW)
  7. intracortical - T - well-demarcated, wedgeshaped “bubbly” intracortical mass in a young patient
  8. degeneration of an anaplastic astrocytoma - F - arises from dysplastic cells in erminal matrix. Extend along migratory path of neurons towards cortex.
  9. cystic change and haemorrhage - F - pseudocystic appearance on T1. Can have microcystic degeneration histologically.
  10. Minimal or no mass effect – T (Osborn)
  11. Majority contain calcification - F - 20-36% contain calcification
159
Q

101.Least common site for meningioma

  1. adjacent to hippocampus
  2. parietal lobes
  3. between cerebrum and cerebellum
  4. adjacent to nose
A
  1. adjacent to nose - F - ectopic (extradural) meningiomas <1% (paranasal sinus, intradiploic space, outer table fo skull, scalp, parotid gland, lung, adrenal).
  2. Least common site for meningioma (TW)
  3. adjacent to hippocampus - T - sphenoid ridge, middle cranial fossa 17-25%
  4. parietal lobes - T - convexity of lateral hemisphere 20-34%
  5. between cerebrum and cerebellum - T - tentorium cerebelli 2-4%
  6. adjacent to nose - F - ectopic (extradural) meningiomas <1% (paranasal sinus, intradiploic space, outer table fo skull, scalp, parotid gland, lung, adrenal).
160
Q

102.Berry aneurysms (T/F):

  1. majority occur at branch points
  2. 40% posterior circulation
  3. Assoc with fibromuscular dysplasia
  4. 30% of ADPKD’s have berry aneuyrsms
A
  1. T
  2. F
  3. T
  4. F
  5. Berry aneurysms (T/F): (TW)
  6. majority occur at branch points - T - vessel bifurcation > lateral wall. Short > long segment.
  7. 40% posterior circulation - F - 90% anterior circulation (IC/PCoA, ACoA most common sites). 10% posterior circulation (BA bifurcation, PICA most common).
  8. Assoc with fibromuscular dysplasia - T - Assoc with hereditary/connective tissue disorders: FMD, ADPKD, Ehlers-Danlos type IV, NF1.
  9. 30% of ADPKD’s have berry aneuyrsms - F - 10%
161
Q

103.Least likely site for hypertensive bleed in the brain is:

  1. hippocampus
  2. cerebellum
  3. basal ganglia
  4. thalamus
  5. pons
A

1.hippocampus 103.Least likely site for hypertensive bleed in the brain is: (TW)

  1. hippocampus
  2. cerebellum - pons, cerebellum 10%
  3. basal ganglia - striatocapsular (putamen/external capsule) 60-65%
  4. thalamus - 15-25%
  5. pons - pons, cerebellum 10%
162
Q

APRIL 2003???104.HSV encephalitis (T/F):

  1. low density medial temporal lobe
  2. sparing basal ganglia
  3. involves cingulate gyrus
  4. enhancement
  5. mass effect
A
  1. T
  2. T
  3. T
  4. T
  5. T
  6. HSV encephalitis (T/F): (TW)
  7. low density medial temporal lobe - T - low attenuation, mild mass effect in medial temporal lobes, insula.
  8. sparing basal ganglia - T - basal ganglia typically spared
  9. involves cingulate gyrus - T - limbic system: temporal lobes, insula, subfrontal area, and cingulate gyrus
  10. enhancement - T - CECT patchy or gyriform enhancement of temporal lobes, late features. T1 C+ may see mild, patchy enhancement early. Gyriform enhancement usually seen 1 week after initial symptoms
  11. mass effect - T - mass effect, edematous brain.
163
Q

105.Sturge-Weber syndrome is associated with the following (T/F).

  1. Contralateral facial lesions with cranial vascular malformation.
  2. Pial venous angiomas.
  3. Tramline calcification.
  4. A V-1 lesion associated with occipital meningeal angioma.
  5. Berry aneurysm.
A
  1. F
  2. T
  3. T
  4. F
  5. F
  6. Sturge-Weber syndrome is associated with the following (T/F) (TW).
  7. Contralateral facial lesions with cranial vascular malformation - F - leptomeningeal angioma occurs in 10-20% of cases when a typical facial lesion is present. The intracerebral lesion usually occurs on the same side as the port wine stain.
  8. Pial venous angiomas - T - pial angiomatosis. angiomatous tissue typically fills the subarachnoid space in the sulci, and large tortuous venous structures drain superficially or inot the deepvenous system.
  9. Tramline calcification - T - tramtrack calcification evident on radiography. Ca++ is in gyral/subcortical white matter. Calcification is not in leptomeningial angioma.
  10. A V-1 lesion associated with occipital meningeal angioma - prob T - CN V1 facial nevus flammeus (Port-wine stain) is most common 98%, +/- V2, V3. Location of angiomatosis unilateral 80%, bilateral 20%. Occipital > parietal > frontal/temporal lobes > diencephalon/midbrain > cerebellum
  11. Berry aneurysm - F - Assoc with hereditary/connective tissue disorders: FMD, ADPKD, Ehlers-Danlos type IV, NF1.
164
Q

106.Epidermoid, which is most correct?

  1. low T1/ high T2
  2. high on ADC
  3. completely attenuates on FLAIR
  4. mild homogeneous enhancement on T1 C+
A
  1. majority low T1/ high T2 - T - “dirty CSF”. Iso- to slightly hyperintense to CSF on T1. Variant of ‘white epidermoid’ has hight T1 signal.
  2. Epidermoid, which is most correct? (TW)
  3. majority low T1/ high T2 - T - “dirty CSF”. Iso- to slightly hyperintense to CSF on T1. Variant of ‘white epidermoid’ has hight T1 signal.
  4. high on ADC - F - low ADC, bright DWI. Restricted diffusion.
  5. completely attenuates on FLAIR - F - do not null (attenuate). “Incomplete attenuation” on FLAIR is suggestive of epidermoid cyst-CPA.
  6. mild homogeneous enhancement on T1 C+ - F - no enhancement is the rule, although mild peripheral enhancement occurs in approx 25% cases.
165
Q
  1. CNS Lymphoma
  2. most unifocal mass
  3. ring enhancement in AIDS
  4. low density on CT
A

2.ring enhancement in AIDS - due to central cavitation

166
Q
  1. Ependymoma
  2. myxopapillary in conus
  3. cellular in thoracic cord
A

both true

  1. myxopapillary in conus
  2. cellular in thoracic cord
167
Q
  1. Carotid angiogram
  2. 2ml/sec 8 mL total appropriate
  3. In AP supraclinoid ICA best shown on waters projection
A
  1. 2ml/sec 8 mL total appropriate
168
Q

110.Extracranial dissection

  1. causes 5-20% of strokes in young people
  2. vertebral artery dissection most occurs between C2 and skull base
  3. carotid artery dissection normally occurs around the bifurcation
  4. Most have underlying vascular disease
A
  1. causes 5-20% of strokes in young people
  2. vertebral artery dissection most occurs between C2 and skull base
  3. Extracranial dissection
  4. causes 5-20% of strokes in young people
  5. vertebral artery dissection most occurs between C2 and skull base
  6. carotid artery dissection normally occurs around the bifurcation
  7. Most have underlying vascular disease
169
Q
  1. Astrocytomas in children. False?
  2. Are the most common posterior fossa tumour.
  3. Calcify more commonly than medulloblastoma.
  4. Are usually a less aggressive lesion.
  5. Usually more cystic than solid.
  6. Show enhancement after contrast administration.
A
  • AJL agree with LW below (I have changed the answers to reflect this)
    113. Astrocytomas in children. (GC)
  1. Are the most common posterior fossa tumour. T
    * LW: TRUE - cerebllar astrocytoma / pilocytic astrocytoma are the most common type of posterio fossa tumour in children (Donnelly).
  2. Calcify more commonly than medulloblastoma F – SG thinks false, roughly equal (Radprimer).
    * LW: FALSE if referring to pilocytic astrocytomas, per Donnelly, cerebellar astrocytomas usually do not show Ca++, unlike medulloblastoma.
  3. Are usually a less aggressive lesion. T – WHO I
  4. Usually more cystic than solid. T
  5. Show enhancement after contrast administration. T – solid components enhance
170
Q
  1. Changes from radiotherapy.
  2. Cannot be distinguished from recurrence on M.R.I.
  3. Can be distinguished from recurrence on PET scans.
  4. Produces necrosis of the brain.
  5. Is reversible.
  6. May show mass effect.
A

true 1, 3, 5

114.Changes from radiotherapy. (GC)

  1. Cannot be distinguished from recurrence on M.R.I. T
  2. Can be distinguished from recurrence on PET scans. F
  3. Produces necrosis of the brain. T
  4. Is reversible. F
  5. May show mass effect. T
171
Q

115.Neurofibromatosis (T/F):

  1. NF1 - pseudoarthrosis is a diagnostic criteria.
  2. NF1 - 10% have some sort of glioma.
  3. NF2 - associated with bilateral acoustic neuromas.
  4. NF2 - have multiple spinal neurofibromas.
  5. Bleed profusely when fractured.
A
  1. F
  2. F
  3. T: NF2 - associated with bilateral acoustic neuromas - T - vestibular schwannomas (previously acoustic neuromas)
  4. F
  5. F
  6. Neurofibromatosis (T/F): (TW)
  7. NF1 - pseudoarthrosis is a diagnostic criteria - F - A distinctive osseous lesion, such as a sphenoid dysplasia or thinning of the long bone cortex, with or without pseudoarthrosis.
  8. NF1 - 10% have some sort of glioma - F - 15-21% have optic pathwya glioma (pilocytic astrocytoma). 10% of all optic nerve gliomas are associated with NF.
  9. NF2 - associated with bilateral acoustic neuromas - T - vestibular schwannomas (previously acoustic neuromas)
  10. NF2 - have multiple spinal neurofibromas - F - schwannomas. Nerve root tumors in NF2 may preset as dumbell-shaped lesions extending through the neural foramina, similar to NF1, but histology reveals that most of these are schwannomas. (skin tags also schwannomas).
172
Q

116.Meningoma (T/F):

  1. 10% are multiple.
  2. The degree of sclerosis with en plaque lesions is directly proportional to infiltration of the bone.
  3. When occur at the C.P. angle, show more homogeneous enhancement than neurofibroma.
  4. Oedema correlates with the size of the lesion.
  5. When intraventricular, have a propensity for the trigone.
A
  1. T: 10% are multiple - T - 1-9% multiple, 16% at autopsy.
  2. F
  3. F
  4. F
  5. T
  6. Meningoma (T/F):
  7. 10% are multiple - T - 1-9% multiple, 16% at autopsy.
  8. The degree of sclerosis with en plaque lesions is directly proportional to infiltration of the bone - F - hyperostosis does not indicate tumor infiltration.
  9. When occur at the C.P. angle, show more homogeneous enhancement than neurofibroma - F - both enhance homogeneously and intensely. (neurofibroma and schwannoma). Dural tail, eccentric to porus acusticus helps Dx.
  10. Oedema correlates with the size of the lesion - F
  11. When intraventricular, have a propensity for the trigone - T - 2-5% occur in ventricles. Of these 80% are in lateral ventricles. Most common trigonal intraventricular mass in adulthood.
173
Q
  1. Miscellaneous.
  2. A technetium lung scan shows positive findings before chest x-ray in PCP.
  3. Leiomyomatosis may produce a painless rib erosion.
  4. Alcohol is good for small vessel bronchial embolisation in haemoptysis.
  5. Mycobacterium avium complex affects peripheral more than abdominal nodes.
  6. Acoustic neuromas more commonly affect the cochlear division.
  7. Otosclerosis is usually bilateral.
  8. M.R.I. is sensitive for early HIV encephalitis.
  9. Toxoplasmosis commonly affects the basal ganglia.
A

1 and 8 true?

174
Q
  1. Cranial nerve lesions.
  2. Osteopetrosis.
  3. Cleidocranial dysostosis.
  4. Morquio’s.
  5. Cavernous sinus thrombosis.
  6. Paget’s disease.
A

probably 1 4, and 5

  1. Cranial nerve lesions.
  2. Osteopetrosis.
  3. Cleidocranial dysostosis.
  4. Morquio’s.
  5. Cavernous sinus thrombosis.
  6. Paget’s disease.
175
Q
  1. The following are features of alobar holoprosencephaly (T/F)
  2. Fused thalami
  3. Third ventricle visible
  4. Absence of corpus callosum
  5. Associated with the Neu Laxova syndrome
  6. Presence of proboscus above the orbits
A
  1. T
  2. F
  3. T
  4. F
  5. T

119.The following are features of alobar holoprosencephaly (T/F): (TW)

  1. Fused thalami - T
  2. Third ventricle visible -F - absence of 3rd ventricle
  3. Absence of corpus callosum - T
  4. Associated with the Neu Laxova syndrome - F - rare condition. IUGR, microcephaly, facial dysmorphy, short neck ,oedema, scaly skin, and perinatal death.
  5. Presence of proboscus above the orbits - T - cyclopia (= midline single orbit); may have proboscis (= fleshy supraorbital prominence) and absent nose.

Absence of septum pellucidum, 3rd ventricle, falx cerebri, interhemispheric fissure, corpus callosum, fornix, optic tracts, olfactory bulb, internal cerebral veins, superior and inferior straight sagittal sinus, vein of galen, tentorium, sylvian fissure, opercular cortex.
Crescent-shaped holoventricle
Severe facial anomalies (“the face predicts the brain”)

176
Q
  1. Intracranial sarcoidosis (T/F)
  2. May be the initial manifestation of the disease
  3. May involve the meninges
  4. Can cause hydrocephalus
  5. The lesions do not enhance with IV contrast
  6. Only rarely causes hypothalamic dysfunction
A
  1. T
  2. T
  3. T
  4. F
  5. F
  6. Intracranial sarcoidosis (T/F) (TW)
  7. May be the initial manifestation of the disease - T - neurologic manifestationsof sarcoidosis occur in approximately 5% of patients with sarcoid. Intracranial sarcoidosis may develop in a patient with known systemic disease or it may be the initial manifestation.
  8. May involve the meninges - T - leptomeninges, dura mater, subarachnoid space, peripheral nerves, brain parenchyma, ventricular system. Affects meninges and cranial nerves more often than the brain.
  9. Can cause hydrocephalus - T - communicating / obstructive hydrocephalus is the most common finding (from arachnoidits / adhesions).
  10. The lesions do not enhance with IV contrast - F - isodense / hyperdense homogeneously enhancing small single/multiple nodules.
  11. Only rarely causes hypothalamic dysfunction - F - depends on what you say is rare. pituitary and hypothalamic dysfunction (eg diabetes insipidus) in 5-10%.
177
Q
  1. Primary intracerebral lymphoma is a recognised complication of
  2. Renal transplantation
  3. Immunoglobulirl A deficiency
  4. Histiocytosis X
  5. Sjogreri s syndrome
A

1.Renal transplantation